Long case examination for phase iii medical students usmkk

89
Long Case Examination for List and Answer to C 200 Muhamad Na’ University Scie r Phase III Medical Students (Obstetr Commonly Asked Questions by Lectur jacknaim@gm www.jacknaim 09/2010 ’im B Ab Razak ence Malaysia ric Cases) rers mail.com msnotes.blogspot.com

description

This booklet in Obstetric is intended for last minute revision before taking a

Transcript of Long case examination for phase iii medical students usmkk

Page 1: Long case examination for phase iii medical students usmkk

Long Case Examination for Phase III Medical List and Answer to Commonly Asked Questions by Lecturers

2009

Muhamad Na’im B Ab Razak

University Science Malaysia

Long Case Examination for Phase III Medical Students (Obstetric Cases)List and Answer to Commonly Asked Questions by Lecturers

[email protected]

www.jacknaimsnotes.blogspot.com

2009/2010

Muhamad Na’im B Ab Razak

University Science Malaysia

(Obstetric Cases) List and Answer to Commonly Asked Questions by Lecturers

[email protected]

www.jacknaimsnotes.blogspot.com

Page 2: Long case examination for phase iii medical students usmkk

Open Notes to My friendsOpen Notes to My friendsOpen Notes to My friendsOpen Notes to My friends

In the name of Allah,

The Most gracious and the Most Merciful:

Assalamu’alaikum wbt,

May Peace and Blessing of GOD be upon all of you

Dear friends,

I would like to take this opportunity to thank you for inspiring me a lot during our journey in

medical school.

This notes is my way of replying your kindness and favor in helping me to survive the

challenging life of medical school. Lot of cry and tears, but yet we still able to laugh together!

All thanks to the seniors who have been doing a great job by compiling the entire

commonly asked question during an exam. I just add some spice to their effort by providing

an answer to the questions through this little book.

If there is any mistake in this book, do not hesitate to inform me. InsyaALLAH I will try my

best to correct it and updating this book.

Whatever you read in this book, please double check with current management and ask

opinion from lecturers.

This book mainly reserves to be used for last minutes revision and not as reference. Please

keep on reading text book and enhance your knowledge through journals. It is our duty as

a Muslim to keep on updating our knowledge

Hopefully, all of us may become a great and outstanding Muslim doctor someday. Pray for

me that I may pass my undergraduate study and successfully pursuit my dream to

become an emergency specialist.

Sincerely yours,

Jacknaim

Page 3: Long case examination for phase iii medical students usmkk

Oath Of USM Medical Student’s Graduation DayOath Of USM Medical Student’s Graduation DayOath Of USM Medical Student’s Graduation DayOath Of USM Medical Student’s Graduation Day

In the name of God,

We seek from you:

The ability to be truthful, honest modest,

Merciful and objective in our dealings

The fortitude to admit our mistakes,

to amend our ways and to forgive

The wisdom to comfort and counsel all our patients

Towards well being, peace and harmony regardless of their

Social status, race and religion

The ability to understand that our profession is sacred,

Dealing with your most precious gifts of life and intellect

We promise to devote our lives in serving mankind,

Poor or rich,

Literate or illiterate,

Irrespective of race and religion

With Patience and tolerance,

With virtue and reverence,

With knowledge and vigilance,

And with Your love in our heart

Page 4: Long case examination for phase iii medical students usmkk

The Oath Of the Muslim DoctorThe Oath Of the Muslim DoctorThe Oath Of the Muslim DoctorThe Oath Of the Muslim Doctor

I swear by God ...The Great

To regard God in carrying out my profession

To protect human life in all stages and under all circumstances,

doing my utmost to rescue it from death, malady, pain and anxiety. .

To keep peoples' dignity, cover their privacies and lock up their secrets...

To be, all the way, an instrument of God's mercy,

Extending my medical care to near and far,

Virtuous and sinner and friend and enemy.

To strive in the pursuit of knowledge and. harnessing it for the benefit

but not the harm of Mankind.

To refer my teacher, teach my junior,

And be brother to members of the Medical Profession.

Joined in piety and charity.

To live my Faith in private and in public,

Avoiding whatever blemishes me in the eyes of God,

His Apostle and my fellow Faithful.

And may God be witness to this Oath.

Page 5: Long case examination for phase iii medical students usmkk

Long Case Examination for Phase III Medical Students University Science Malaysia

Important and Common Cases Needs to be Covered in Obstetric Section.

1) Normal labour

2) False labour

3) Unsure of Date

4) Induction of Labour

5) Caesarian Section

6) Pregnancy Induce Hypertension

7) Pre eclampsia

8) Hypertension in Pregnancy

9) Diabetes Mellitus in pregnancy

10) Gestational Diabetes Mellitus

11) Oligohydramnios

12) Polyhydramnios

13) Reduced Fetal Movement

14) Threatened pre term labour

15) Premature birth.

16) Post Date

17) Post Term

18) PPROM

19) PROM

20) Placenta previa

21) Unstable lie

22) Breech presentation

23) Multiple pregnancy

24) Heart disease in pregnancy

25) Anemia in pregnancy

26) Fibroids

27) Anti phospholipids syndrome

28) Teenage pregnancy

And We have enjoined on man (to be good) to his parents: in travail upon travail did his

mother bear him, and in years twain was his weaning: (hear the command), "Show

gratitude to Me and to thy parents: to Me is (thy final) Goal [Q31:14]]

Page 6: Long case examination for phase iii medical students usmkk

Long Case Examination for Phase III Medical Students University Science Malaysia

Anti tetanus toxoid Type of immune (page 36) When to give (page 36) Anemia in pregnancy (page 44,45) Breech presentation (page 36) Causes and complication Mode of delivery Candidosis Drug (page 24) Caesarian section (page 9) Anterior abdominal wall layer (page 10) Impending scar rupture (page 10) Preparation pre op and post op (page 9) Pfannensteil scar (page 32) Trial of scar (page 9) Cervix Normal cervical length (page 3) cervical effacement (page 1) cervical dilatation in nulli vs multiparity (page 1) Cervical cerclage (page 26) Chorioamnionitis Common organism (page 8) Management (page 8) Diabetes Melitus in pregnancy GDM (page 15) Complication of GDM (page 16) Diagnosis and level of sugar control (page 15) Screening test (page 15) Diabetogenic hormone (page 15) Hydrocephalus (page 19)

Macrosomic baby (page 18) Management (page 16, 17) MOGTT, when to do (page 15) MOGTT, Indication (page 16) Shoulder dystocia (page 18) Spontaneous vaginal delivery ((page 18) Weight gain in pregnancy (page 15) Heart disease in pregnancy (page 40, 41, 42, 43) Aspirin, IV Contraindication for pregnancy (page 43) Failure (page 40) Eisenmenger syndrome (page 41) Warfarin (page 41) Episiotomy Definition (page 4) Layer cut (page 4) Fibroids (page 46) Hypothyroidism (page 50) Labour Bishop score (page 1, page 3) Braxton Hicks contraction (page 2) definition of labour (page 5) Discharging patient in latent phase of labour (page 3) engagement (page 4) Show (page 5) True vs false labour (page 2) Management of active phase of labour (page 2) Mechanism of labour (page 4) Induction of labour definition (page 7) Indication (page 7)

Page 7: Long case examination for phase iii medical students usmkk

Long Case Examination for Phase III Medical Students University Science Malaysia

Mehtod (page 7) Oligohydramnios (page 20) Parity Pseudoprimid (page 5) Grand multiparity (page 26) Great grand multiparity (page 49) Polyhydramnios (page 21) Post date (page 27) Post term (page 28, 29) PPROM (page 30) Fever Positive findings Management PROM (page 31) Placenta seperation, sign and symptoms (page 2) Placenta previa (page 32, 33) Prostin Complication (page 7) dose (page 8) Instruction to patient (page 8) with presence of contaction pain (page 1, page 3) Pregnancy induced hypertension definition (page 11) Essential hypertension (page 14) Management (page 11) Pre eclampsia (page 12) Impending eclampsia (page 13) Magnesium sulphate (page 13)

Ultrasound placenta (page 6) Premature labour (page 24) Definition Premature contraction (page 25) Management Reduce fetal movement (page 22, 23) Tocolytic Fetal kick chart Tradisional medicine (page 23) Twin pregnancy (page 37, 38, 39) Classification complication Physical examination Twin to twin transfusion reaction Unsure of date (page 7) Neagele's rule (page 7) Comfirmation of date Unstable lie (page 34, 35) Causes Complication Management Mode of delivery Uterus support (page 4)

Page 8: Long case examination for phase iii medical students usmkk

Long Case Examination for Phase III Medical Students University Science Malaysia

1

25 years old Malay lady, G1P0 at 37W+ 2/7 are admitted because of contraction pain but not associated with show or leaking. Questions

1) How do you access the favorable of cervix?

2) What is cervical effacement? 3) Are there any differences if the cervix is

1 cm dilated in primid vs. Multipara who presented with contraction pain at term?

4) Can we induce the labour with Prostin with the present of recorded contraction pain?

Answer

Cervical score In HUSM, we used Modified Bishop Score. Cervix is favorable if Bishop score > 5

Mnemonics: DiCoLePoS (Dilatation, consistency, length, Position and Station) [Credited to Dr Ramli Ibrahim, HUSM]

Cervical Effacement Cervical changes prior to onset of labour where cervix become shorter, softer and moves from its position in the posterior vaginal fornix towards anterior vaginal fornix [Joan Pitkin et al, Obstetrics and Gynaecology: An Illustrated colour Text]

Image from: Joan Pitkin et al, Obstetrics and

Gynaecology: An Illustrated colour Text

Differences if the cervix is 1 cm dilated in primid vs. Multipara who presented with

contraction pain at term? Nulliparous women have small external os at cervix center. In multiparous woman, cervix is bulkier and the external os has a more slit like appearance. Therefore, dilatation of 1 cm is significant in primid and not in multigravida. In multiparous, it is usually normal if cervix dilatation is 1 cm. Diagnosis of false labour should be made if it did not progress.

Role of Prostin in the presence of recorded contraction pain.

Recorded contraction pain is by evidence of CTG reading plus typical history of contracting pain. Once it present, Prostin should never being use as it will predispose mother to uterine hyper stimulation and cause fetal distress to the baby. Other mode of induction of labour should be considered.

Page 9: Long case examination for phase iii medical students usmkk

Long Case Examination for Phase III Medical Students University Science Malaysia

2

Case: G1P0, Post EDD, currently in labour (VE 5cm) Question:

a) Patient is a primid, never experience before, how are you going to ask her in Hx whether it is a true labour.

b) How to manage this patient Notes: A Braxton Hicks contraction is a normal irregular uterine contraction starts occurring from fourth months of pregnancy. It acts as preparation for uterus to contract properly later.

True vs. False Labour pain 1) Timing of contractions False Labor: Often are irregular and do not

get closer together True Labor: Come at regular intervals and as

time goes on, get closer together. Contractions last about 30-70 seconds.

2) Change with movement False Labor: contractions stop in association

with walking or change in position. True Labor: Contractions continue, despite

movement or changing positions

3) Strength of contractions False Labor: Contractions are usually weak

and do not get much stronger (may be strong first, then get weaker)

True Labor: Contractions steadily increase in strength

4) Pain of contractions False Labor: contractions are usually only

felt in the front of the abdomen or pelvic region True Labor: Contractions usually start in the

lower back and move to the front of the abdomen. Referred pain from uterus felt at the buttock.

Management of this patient

Impression: Patient is already in active phase of first stage of labour. Post date patient who are already in labour will not change the management and spontaneous vaginal delivery should be expected except there is indication for caesarian section or instrumental deliveries. General management

1) Transfer patient to Labour room 2) FBC and GSH

First stage of labour

1) Review history and problems 2) Abdominal exam and VE +ARM 3) Starts partogram 4) Review patient after 4H since cervix is

<6cm (if >6cm VE when full dilatation is expected)

5) Monitor a) Maternal BP, PR, Uterine

contraction b) 4H temperature c) FHR auscultation/ CTG

Second stage of labour

1) Leave patient for 30 minutes if no pushing contraction. Notify MO if not deliver after 1H of active pushing

2) Episiotomy Third stage of labour (30 Minutes)

1) Syntometrine (Oxytoxin 5U+ ergometrine 0.5 mg) IM

2) Delivery of placenta by controlled cord traction

3) Repairing of episiotomy wounds

Signs of placenta separation - Uterus contract and fundus become

globular and firm - Small gush of blood flow out - Lengthening of umbilical cord.

Page 10: Long case examination for phase iii medical students usmkk

Long Case Examination for Phase III Medical Students University Science Malaysia

3

28 years old Malay lady, G1P0 at 38W + 5/7 days POA was admitted because of contraction pain. There is no show or leaking liquar.Below is her Bishop score on admission.

Questions

1) Comment on the Bishop score 2) What is the normal length of the cervix

in non pregnant lady? 3) If this patient requested to be

discharged, can you allow that? Support your answer.

4) Will you induce this patient for labour?

Bishop Score

Based on assessment on Bishop Score, patient is already in latent phase of labour as evidence of os is dilated. However the cervix is not favorable for labour yet.

Normal cervix length for non pregnant lady 3.5 CM

Requested to be discharged This patient is in the latent phase of labour. In primid, the latent phase could be as short as one day but may extend up to one week.

Before, any decision to discharge this patient, few measures needs to be look at.

1) If the contraction pain starts to subside 2) Pre discharge vagina examination did

not show any cervical progression 3) No Pre labour ruptures of membrane. 4) CTG has been performed and reactive 5) Baby is not in mal presentation. 6) Patient can easily come back to hospital

if anything happen. a) Short distance b) Access to transportation c) People to take care of her.

7) With advice that patient must come to hospital if any PROM or show or if the contraction become strong and in close intervals.

Induction of labour

- Induction with prostin is not indicated as

contraction is already there. It will only increase risk for uterine hyper stimulation and abruptio placenta.

- ARM could be done if cervical dilatation more than 3 cm.

Page 11: Long case examination for phase iii medical students usmkk

Long Case Examination for Phase III Medical Students University Science Malaysia

4

23 years old Malay lady at 39W+3/7 POA was transferred into ward from labour room because of contraction pain associated with show on the day of admission. No leaking of liquor reported and fetal movement was good. Questions

1) Types of pelvis 2) What is engagement 3) Outline the mechanism of labour 4) What is the layer cut during the

episiotomy procedure? 5) The structures supporting the uterus.

Engagement

Descent of the biparietal diameter of the fetal head below the plane of the pelvic inlet. Clinically, if the lowest portion of the occiput is at or below the level of the maternal ischial spines (station 0), engagement has usually taken place. Engagement can occur before the onset of true labor, especially in nulliparous patients [The John Hopkins Manual of Gynecology and Obstetrics 3rd ed.]

Mechanism of labour

Changes in position of the fetal head during passage through the birth canal in the vertex presentation. (EDFIERE!)

1) Engagement 2) Descent 3) Flexion 4) Internal rotation 5) Extension 6) External rotation 7) Expulsion

What is episiotomy and layer cut during the procedures?

Episiotomy is a surgical cut that is made to the perineum during the pushing stage of labour. Layers of cutting:

1) Skin 2) Subcutaneous tissue 3) Vaginal mucosa 4) Bulbospongiosus muscle 5) Deep and superficial transverse perineal

muscle

Support of the Uterus

1) Tone of levator ani muscle 2) Perineal body 3) Ligaments

a. Transverse cervical or cardinal ligament

b. Pubocervical c. Sacrocervical

Page 12: Long case examination for phase iii medical students usmkk

Long Case Examination for Phase III Medical Students University Science Malaysia

5

36 years old Malay lady, G3P0 at 37 weeks of pregnancy was admitted to ward after noticing spotting blood mixing with mucous on her underpants after waking up from sleep. The same event occurs two times in ward. However, there is no recorded abdominal pain. Question

1) What is mature pseudo primid? 2) What is labour 3) Terminology for blood mixing with

mucous 4) Differential diagnosis 5) Management to this patient.

Answer

Mature pseudo primid - Mature means age of mother > 35 years

old - Pseudo primid means patient has been

pregnant but never deliver the baby. - The term ‘mature’ should alert the

doctor in carefully managing this patient because of many complication can occur in this age group. Furthermore, this could be her last pregnancy

Labour

- Process by which fetus is expelled from the uterus and into the outside world.

- Three stages of labour a) 1st stage- onset of contraction till

full dilatation of cervix b) 2nd stage- full dilatation of cervix till

delivery of fetus c) 3rd stage- delivery of placenta

- Sign and symptoms of labour includes abdominal contracting pain, show (discharging blood mixing with mucous), gushing of clear fluid (liquor)

Differential diagnosis - In labour - False labour - PROM - Bleeding from PP or Placenta abruptio. - Discharge from urinary tract infection - Trauma to the perineal region.

Management to this patient

1) Full history and physical examination

a) Correct dating of pregnancy b) Elicit any risky pregnancy c) Eliminating the differential

diagnosis. d) It is important to exclude PROM as

patient at term and chorioamnionitis could be disastrous for fetus.

2) Observation of vital sign 3) Fetal kick chart (some doctor

recommend this) and labour progression chart (LPC)

4) Speculum examination to access the cervix and excludes PROM, infection.

5) Assessment of fetal well being (CTG and ultrasound)

6) Blood investigation (FBC to look for evidence of infection)

7) Urine FEME to exclude UTI. 8) Observe the patient in wards for 2-3

days. If patient is stable and the labour does not progress, then the diagnosis is false labour. Patient can be safely discharge and ask her to come back again once the sign and symptoms of labor starts.

9) If patient in labour, then proceed with the management for labour

Page 13: Long case examination for phase iii medical students usmkk

Long Case Examination for Phase III Medical Students University Science Malaysia

6

Notes on U/S about placenta

Vascularity

Very vascular – has 2 blood supplies

Blood from fetus through 2 (sometimes 1) umbilical arteries through umbilical cord from fetal hypogastric arteries to placenta

1 umbilical vein carries blood back to fetal left portal vein

Blood from mom through branches of uterine arteries through the myometrium (arcuate arteries) through the basilar plate (spiral arteries) into the placenta

The two circulations intertwine in the placenta but do not mix

Exchange of oxygen and nutrients occurs over the large vascular surface area

Maternal venous channels in the placenta are hypoechoic or anechoic spaces called venous lakes (usually small, but can be large)

Anatomy on US

Inner border of placenta against the uterine wall has the combined hypoechoic myometrium and interposed basilar layer = hypoechoic band called the decidua basalis (contains maternal blood vessels)

Outer surface abutting the amniotic fluid = chorionic plate (chorioamniotic membrane) = bright specular reflector

Placental thickness judged subjectively

But if measure at midposition or cord insertion 2-4 cm = normal

Grade 0 1.Late 1st trimester-early 2nd trimester 2.Uniform moderate echogenicity 3.Smooth chorionic plate without indentations Grade 1 1.Mid 2nd trimester –early 3rd trimester (~18-29 wks) 2.Subtle indentations of chorionic plate 3.Small, diffuse calcifications (hyperechoic) randomly dispersed in placenta Grade 2 1.Late 3rd trimester (~30 wks to delivery) 2.Larger indentations along chorionic plate 3.Larger calcifications in a “dot-dash” configuration along the basilar plate Grade 3 1.39 wks – post dates 2.Complete indentations of chorionic plate through to the basilar plate creating “cotyledons” (portions of placenta separated by the indentations) 3.More irregular calcifications with significant shadowing 4.May signify placental dysmaturity which can cause IUGR 5.Associated with smoking, chronic hypertension, SLE, diabetes Sources: http://www.learningradiology.com/notes/gunotes/placentapage.htm

Page 14: Long case examination for phase iii medical students usmkk

Long Case Examination for Phase III Medical Students University Science Malaysia

7

Case: Unsure LMP/ Unsure of Date Questions

a) Neagele’s rule b) Investigation c) Management d) Induction of labour e) Complication of Prostin

Neagele’s rule

1) Sure of date 2) Menstrual cycle is regular of 28 days

(ovulation occur 14 days prior to the next menses)

3) Not on any form of hormonal contraception within 3 months

4) Not lactating within 2 months

Investigation 1) Cardiatocography (CTG) to access fetal

well being 2) Ultrasound for physical biometry of the

baby, and amniotic fluid index

Management

1) Confirmation of the date of pregnancy a) Early ultrasound scan (<20w) b) 1st UPT positive (6-8w) c) Quickening d) Uterine size correspond to

pregnancy e) Onset of signs and symptom of

pregnancy f) Conception date

2) Bishop score (>5 is favorable) 3) Elicit any medical problem.

Induction of labour

An intervention designed to artificially initiate uterine contractions leading to progressive dilatation and effacement of the cervix and the birth of the baby. The term is usually restricted

to pregnancies at gestations greater than the legal definition of fetal viability (24 weeks). Divided into mechanical (Sweep & scratch, ARM,) and pharmacological (IV Syntocinon, Prostin). Others; breast stimulation, relaxin, hyaluronidase, sexual intercourse, acupuncture, homeopathic method

Indication for IOL 1) Fetal

a) IUGR b) PIH/PE c) GDM at 38w d) Post EDD e) Twin at term f) Hx of unexplained APH g) Transverse oblique/unstable lie h) Hemolytic disease i) Fetal abnormality incompatible with

life (anencephaly) 2) Maternal

a) Medical disorder aggravated by pregnancy like DM, SLE, PE, Renal disease.

b) IUD with risk of DIC c) Spontaneous/ PROM>24h d) Abruption of placenta

Complication of prostin 1) Failed IOL (require c-sec) 2) Uterine hyper stimulation 3) Uterine rupture. 4) Fetal distress. 5) C/I in patient with asthma/glaucoma 6) Abruptio placenta

Page 15: Long case examination for phase iii medical students usmkk

Long Case Examination for Phase III Medical Students University Science Malaysia

8

27 years old Malay lady, G2P1 at 38 weeks of pregnancy was admitted to ward because of PROM more than 24 hours. She was council for induction of labour. Question

1) Common organism causing chorioamnionitis in PROM and how to manage.

2) Dose of Prostin 3) Instruction to the patient before inserting

the Prostin Answer Common organism causing chorioamnionitis

in PROM and how to manage?

1) Risk of getting infection arises after 12 hours of PROM.

2) Antibiotic prophylaxis should be given based on common isolated organism which is group B Streptococcus (IV Penicillin)

3) Chorioamnionitis is more dangerous to fetus as compared to mother

4) IOL should be suggested to the mother if PROM > 24 hours. 90% of patient with ruptured membrane will deliver the baby within 24 hours.

5) If chorioamnionitis develop, patient should be covered with antibiotic against GBS, gram negative and anaerobes.

Dose of Prostin

Notes: I suppositories equals to 3 mg. In primid, we can insert 1 suppository and access the Bishop score 6 hours later. If cervix is favorable, then we may proceed with artificial rupture of membrane. If not, second dose of Prostin may be given.

In woman whom deliver more than two babies (not grand Multipara) and 1 caesarean section scar, the dose for each cycle is 1.5 mg. If labour is not progress after the second dose, then it is considered as failed induction [NICE Guidelines] and emergency C-sec will be done. HUSM did not follow this guidelines and IOL with prostin is based on clinical experience. Some might consider failed induction after the third dose, 6 hours after the second dose. (Controversy) Notes: Prostin is contraindicated if presence of uterine contraction to avoid uterine hyper stimulation.

Instruction to patient before inserting the

Prostin

1) Ask the patient to urinate first because she needs to lie on bed for one hour

2) Ask the patient to lie down on bed for one hour

3) Ask the patient to inform the doctor if the contraction pain is strong.

4) Do CTG after one hour to access uterine contraction and any evidence of fetal distress

5) After one hour, do the VE to access the cervical dilatation.

6) If cervix is more than 3 cm, remove the residual Prostin and sent patient to labour room.

7) If less than that, and suspect uterine hyper stimulation, remove the residue Prostin as well and send patient to labour room and monitor with CTG. KIV tocolytic agents (salbutamol). If fetal distress, emergency cs.

8) If CTG normal, patient can regain her activity. Recheck cervical score 6 hour later.

Page 16: Long case examination for phase iii medical students usmkk

Long Case Examination for Phase III Medical Students University Science Malaysia

9

Case: 3 Previous C-sec scars Question

a) Investigation b) Management c) Post op-acute management

Trial of Scar

Notes: According to ACOG guidelines on vaginal birth after Caeserean Section, trial of scar is not recommended in patients at high risk for uterine rupture. One of the contraindication including this case.

1) Two prior uterine scars and no vaginal deliveries

2) Previous classical or T-shaped incision or extensive transfundal uterine surgery

3) Previous uterine rupture 4) Medical or obstetric complication that

precludes vaginal delivery 5) Inability to perform emergency cesarean

delivery because of unavailable surgeon, anesthesia, sufficient staff, or facility

Notes: The management of this patient should emphasize more on caesarean section and anticipating in possibility of uterine rupture. It also includes advice for tubal ligation. (Practice in Malaysia to do BTL after 4 Caesarian Section) Investigation Fetal investigation

- Ultrasound (AFI, Estimated fetal weight, exclude placenta previa, accrete or abruptio, biometry)

- CTG Maternal (preparation for C-sec)

1) For patient in labour (fluid diet and T. Ranitidine 150 mg q.d.s)

2) Patient at high risk of anesthetic( sips of water+ IV fluid if indicated)

3) To cover the surgery a) Consent form signed b) Baseline blood investigation (FBC,

GSH, LFT, BUSE/Creat) c) Blood cross match (2U Pack cell) d) IV ampicillin 1g stat for

prophylactic e) Bladder catheterization f) Pre med (IV Ranitidine 50mg in 10

ml by slow injection, IV Maxalon 10 mg by slow injection, Sodium citrate 30 ml orally)

4) Anesthetist with at least one year experience

5) Ideally use regional block except contra indicated (major placenta previa, local skin sepsis, severe heart disease, coagulation disorder, severe fetal distress, cord prolapsed, eclampsia)

6) Present of obstetrician. 7) Reduce risk of thromboembolic

phenomenon after surgery a) Early ambulation b) Anti embolic stocking/Flowtron c) Anti coagulant for high risk cases.

[The practical Labour Suite Management- Dr Adibah Ibrahim]

Post op management 1) Recovery area (one to one observation until patient has airway control, cardio respiratory stability and can communicate) 2) In wards (1/2hly observation RR, HR, BP, pain and sedation) for 2H, then hourly if stable 3) Intrathecal opiods- hourly observation for RR, Sedation and pain scores for 12h for diamorphines and 24h for morphines) 4) For epidural opiods and patient-controlled analgesia with opiods (hourly monitoring during CS, plus 2h after discontinuation) 5) Post natal care (analgesic, monitor wound healing, signs of infection) 6) consider CS complication (endometritis, thromboembolism, UTI, urinary tract trauma) [NICE Guidelines on Caesarian Section]

Page 17: Long case examination for phase iii medical students usmkk

Long Case Examination for Phase III Medical Students University Science Malaysia

10

26 years old Malay lady, housewife, G2P1 at 38 weeks of gestation with second husband and history of previous caesarean section was admitted because of c-sec scar tenderness. Questions

1) S&S of impending scar rupture 2) Management for patient come with

impending scar rupture 3) Elicit the scar tenderness on PE 4) The anterior abdominal wall layer cut

during the c-sec operation.

Uterine scar rupture

A complete uterine rupture is a tear through the thickness of the uterine wall at the site of a prior cesarean incision. Patient might present with:

1. Fetal distress evidence by abnormalities in fetal heart rate

2. Vaginal bleeding 3. Sharp onset of pain at the site of

previous scar 4. Sharp pain between contractions 5. contraction become less intense (finally

lead to atony) 6. Diminished baseline uterine pressure 7. Abdominal tenderness 8. Recession of the presenting fetal part 9. Hemorrhage 10. shock

Management to impending scar ruptures

Management Prophylactic management 1) Close monitoring for woman with high risk of uterine rupture 2) Early detection of causes of obstructed labour 3) Use Oxytoxin with caution

4) Forceps application and breech extraction once full cervical dilatation achieves 5) Elective caesarean section 6) Explore the genital tract after difficult or instrumental delivery 7) Blood FBC and GSH Once the ruptured occur 1) Secure the ABC. 02 100%, 3L/min increase oxygenation to tissue if hemorrhage occurs. 2) 2 large bore IV line 3) Blood transfusion and shock management 4) Emergency laparatomy 5) Delivery of fetus and placenta 6) Exploration of the rupture site a) Try to repair the lesion b) Hysterectomy of not salvageable 7) Internal iliac artery ligation in case of broad ligament hematoma because uterine artery is usually retracted and difficult to be identified. 8) Vaginal repair if there is cervical tear

Layer cut through caesarean section (Pfannenstiel approach)

1) Curved transverse cut just below hair

border a) skin b) superficial fascia (Camper and

Scarpa) c) Rectus sheath (contains fascia of

EO, IO and TM)

2) Vertical incision for access into lower abdomen a) Separation of rectus abdominis

muscle in midline b) Dividing of the fascia transversalis c) periperitoneal fat tissue d) peritoneum

Page 18: Long case examination for phase iii medical students usmkk

Long Case Examination for Phase III Medical Students University Science Malaysia

11

Case: PIH Questions

a) Differential diagnosis b) Management c) Drugs (SE&MOA) d) Drugs contraindicated in PIH

Definition

BP more than or equal to 140/90 mmHg in previously normotensive patient, @ A rise in systolic BP of > 30 mmHg or diastolic BP > 15 mmHg compared with pre-conception or first trimester value in two recording of at least 4H apart

Differential diagnosis - Chronic hypertension (long or before 20w) - Pre eclampsia (>20W+new onset proteinuria) - PE with superimposed chronic HPT New onset or A) acutely worsen proteinuria, B) sudden increase in BP, C) thrombocytopenia or D) elevated liver enzymes after 20 week gestation in women with pre existing HPT - Gestational HPT (after 20w without proteinuria)

Management 1) if detected <20W, must exclude molar

pregnancy by US and after exclusion, being investigate for primary or secondary HPT

2) If pre existing HPT during Booking, should be managed by obs+internist

3) Every other day BP check at local clinic if BP is first high during any ante natal check up.

4) Investigation for PE profile (platelet count, uric acid, serum creatinine level, AST, urine albumin). If PE is diagnosed, then it should be repeated once a week

5) If BP sustained at >100mg/ >25 increment mmHg or clinical suspicious of IUGR, poor maternal-feternal well

being, abnormal surveillance basic blood test (BP and urine dipstick at least 3X per week, weekly PE profile and CTG.)

6) Starts anti hypertensive when diastolic BP > 90 mmHg a) T. Methyldopa 250 mg tds to max

dose of 3g/day or b) T. labetolol 100 mg tds to max 300

mg tds 7) IM dexamethasone 12 MG 12 hourly for

two doses for expectant prem delivery. In case of severe PE

1) Manage in hospitals 2) Close monitor BP 4Hly, reflex, clonus 3) Check fundus 4) Twice weekly(or more based on

severity) PE, CTG, biophysical profile and doppler

5) Anti-hypertensive but aim for 20-25 reduction only and not normal by using hydrallazine or labetolol

In labour 1) BP stabilization 2) Watch for fluid overload (monitor UO) 3) Seizure prophylaxis in severe PE 4) Epidural analgesic is the best 5) Oxytoxin only to augment labour. 6) Never allow woman with severe PE to

push excessively. If BP high, consider instrumental delivery.

7) C/I to ergometrine/syntometrine in third stage due to hypertensive effect.

Drugs contraindicated for PIH includes ACE inhibitor and ARB as it can cause renal dysgenesis of the baby.

Page 19: Long case examination for phase iii medical students usmkk

Long Case Examination for Phase III Medical Students University Science Malaysia

12

Case: 41/M/F, G1P0 at 29W+2d POA High blood pressure and proteinuria 3+ Question

a) Investigation and reason b) Treatment plan c) Time of delivery and why?

My impression: High blood pressure with proteinuria could lead to Pre eclampsia which is worrisome due to serious complication. Therefore, PE should be ruled out first before considering other condition that may falsely give positive result to proteinuria like UTI PE is defined as: Hypertension unique to pregnancy, diagnosed after 20W of gestation and associated with new onset proteinuria; Eclampsia if seizure occurs. If woman already having pre existing HPT but after 20W she develops new onset proteinuria, sudden increase in BP, thrombocytopenia or elevated liver enzymes, then PE with superimposed on chronic hypertension must be suspected. HELLP (Hemolysis, Elevated liver enzyme, low platelet) is a variant of PE with involvement of liver giving rise to tender epigastric pain, and finally DIC.

Investigation 1) Repeat Dipstick testing within 6H

PE shows by urinary albumin >300mg/24 hour@ >1g/l in 2 random urine 6 hour a part. 1+ = 0.3 g/l, 2+ = 1 g/l and 3+ = 3 g/l.

2) 24 Hour proteinuria to see severity of PE. Severe PE >5000mg/24 hr.

3) BP should be checked every 15 minutes until women are stable. Then,

4) Close monitoring of BP (at least 4Hourly) + reflex, clonus.

5) PE profile twice a week (severe PE) or once a week(mild PE) compose of a) Platelet count (decrease) b) Uric acid (1st indication of renal

impairment) c) Sr Creatinine level (renal function) d) Liver enzyme, AST (liver damage) e) Urine albumin as mention in above.

6) Clotting study if platelet < 100 x 106/l 7) Input/output Fluid Chart. 8) CTG for fetal well being. 9) Serial ultrasound measurements of fetal

size, umbilical artery Doppler and liquor volume

Treatment plan

Mild PE T. Methyldopa 250mg tds, max 3g/day or T. Labetolol 100 mg tds, max 300mg tds Or, Tab. Nifedipine 10 mg tds stat dose Severe PE IV hydrallazine start 5mg, double if no effect until 35mg. change drug if fails or IV Labetolol start 10 mg, double if no effect until max 300mg/day) ** MgSo4 slow infusion 4g 10-15 minutes. Maintenance dose IV ig/hour

When to Deliver 1. Delivery is definitive treatment if mother life is compromised. (Very high uncontrolled BP, platelet <100, AST>150 iU/L 2. Can wait until term if well controlled and fetal is not compromised. 3. If gestation >34W, then delivery after stabilization is recommended 4. In this case, prolong delivery for 24 Hr to give steroid injection for lung maturity [RCOG Guideline No. 10(A) March 06]

Page 20: Long case examination for phase iii medical students usmkk

Long Case ExaminationUniversity Science Malaysia

Case: 19/M/F, G1P0 at 32W of pregnancy diagnosed with pre eclampsia at 28W ofgestation. Question

a) Signs and symptoms of impending eclampsia

b) Magnesium sulphate

Signs and symptoms of impending eclampsia1) Headache 2) N & V 3) Visual Disturbances 4) Right upper quadrant pain 5) Progressively oedema (non dependant)6) Frothy urine (proteinuria

Magnesium sulphate

Magnesium sulfate is superior to other AED (phenytoin, diazepam). Indications:

1) Eclampsia 2) Fulminating severe PE either:a) Severe hypertension (MAP: >125

mmHg, SBP: >170 mmHg, DBP: >110 mmHg); OR

b) Hypertension with proteinuria (BP: >180/90 mmHg, proteinuria: >0.3g/24h), AND one of the following:

i. Clonus (>3 beats) ii. Severe persistent headacheiii. Visual disturbance iv. Epigastric pain v. Platelet count <100 x 103/dL

Protocol for use of Magnesium Sulfate:(5ml vial contain 2.5g MgSO4 ~0.5g/ml) Loading Dose – 4g Magnesium Sulfate8ml (4g) + 12ml 0.9% saline IV over minimum of 10 - 15 minutes Maintenance Dose

Long Case Examination for Phase III Medical StudentsUniversity Science Malaysia

13

Case: 19/M/F, G1P0 at 32W of pregnancy diagnosed with pre eclampsia at 28W of

Signs and symptoms of impending

Signs and symptoms of impending eclampsia

Progressively oedema (non dependant)

Magnesium sulfate is superior to other AED

Fulminating severe PE either: Severe hypertension (MAP: >125 mmHg, SBP: >170 mmHg, DBP: >110

Hypertension with proteinuria (BP: mmHg, proteinuria:

>0.3g/24h), AND one of the following:

Severe persistent headache

Platelet count <100 x 103/dL

Protocol for use of Magnesium Sulfate: (5ml vial contain 2.5g MgSO4 ~0.5g/ml)

4g Magnesium Sulfate 8ml (4g) + 12ml 0.9% saline IV over minimum

1g/h for at least 24h after last seizure or deliveryAdd 4 vials (10g) to 50cc of normal saline & run at 5cc/h If further fits occur give a furthof 2g & continue the maintenance infusion Contraindications for Magnesium Sulfate:Cardiac failure Acute renal failure Drug monitoring: Clinical 1) Patellar reflex: - After completion of loading dose - Half hourly whilst on maintenanc - use elbow reflex if epidural in situ2) Respiratory rate: should be >16/min3) Hourly urine output: should be >25ml/h (urine output is critical as serum Mg level depends on renal excretion) 4) Pulse Oximetry : must remain >90% Serum Mg level should be checked when:Oliguria (<25ml/h) Respiratory rate <16/min Pulse oximetry <90% Continuing fit Toxicity (therapeutic range: 2mg/dl) Loss of patella reflex Weakness Nausea Feeling of warmth Flushing Double vision Slurred speech Muscle paralysis Respiratory arrest Cardiac arrest >12 mmol/l[Labor suite Management by Dr Adibah Ibrahim]

for Phase III Medical Students

1g/h for at least 24h after last seizure or delivery Add 4 vials (10g) to 50cc of normal saline & run

If further fits occur give a further slow IV dose of 2g & continue the maintenance infusion

Contraindications for Magnesium Sulfate:

After completion of loading dose Half hourly whilst on maintenance infusion use elbow reflex if epidural in situ Respiratory rate: should be >16/min Hourly urine output: should be >25ml/h

(urine output is critical as serum Mg level

Pulse Oximetry : must remain >90%

level should be checked when:

(therapeutic range: 2-4 mmol/l @ 4-8

5mmol/l

6-7 mmo/l

>12 mmol/l [Labor suite Management by Dr Adibah Ibrahim]

Page 21: Long case examination for phase iii medical students usmkk

Long Case Examination for Phase III Medical Students University Science Malaysia

14

Case: 34/M/L, G2P1 C/C-High blood pressure Dx- Essential hypertension. Question: Hx and Pe only Essential hypertension -Primary elevation of blood pressure without known causes which can be ameliorated only by lifelong pharmacological therapy [Kumar& Clark 6th edition] Risk factor

- Genetic - Low birth weight - Environmental factor

a) Obesity b) Alcohol intake c) Sodium intake d) Stress e) Smoking

- Humoral mechanism (insulin resistance) Cardiac output rises in pregnancy, however there is relative greater fall in peripheral resistance, therefore BP in pregnant woman is usually low than those not pregnant [Kumar& Clark 6th edition]

Important history to be elicited 1) Risk factor to develop pre eclampsia

a. existing chronic medical disorders such as obesity, hypertension, diabetes mellitus, renal disease, connective tissue disease and thrombophilia,

b. Previous history of preeclampsia or eclampsia or IUGR or unexplained stillbirth

c. Family history of preeclampsia or eclampsia, and

d. Extremes of reproductive age (below 20 or above 40 years old)

Investigation 1) ECG 2) Urine dipstick test 3) Fasting Lipid profile 4) BUSE and creatinine,

Management

Non pharmacological

1. Lifestyle medication with light exercise. 2. Reduce the intake of salt and fat.

Pharmacological

1. Stop ACE inhibitor and ARBs. Atenolol can cause IUGR and Labetolol is relatively contraindicated in Asthmatic patient.

2. T. Methyldopa 250mg tds, max 3g/day or

3. T. Labetolol 100 mg tds, max 300mg tds or

4. Tab. Nifedipine 10 mg tds stat dose ** Do not give Methyl dopa together with Nifedipine.

5. High calcium supplementation of 1.5 g/day to prevent PE

6. Avoid Combined vitamins C and E (in the form of tocopherol from soybean) as it may cause IUGR

Others measurement

1. Routine ante natal check up. 2. Advise patient to come immediately to

hospital if develop signs and symptoms of impending PE.

3. Urinary Dipstick to screen new onset of proteinuria.

4. CTG and ultrasound to monitor fetal well being.

5. Re assurance to the patient. 6. Can allow delivering via SVD unless

there is indication for C-Sec.

Page 22: Long case examination for phase iii medical students usmkk

Long Case Examination for Phase III Medical Students University Science Malaysia

15

25 Years Old Malay lady, Nurse, G1P0 at date + 5/7 was admitted to wards because of contraction pain and URTI. Patient also was investigated for GDM because of excessive weight gain during 21 week of pregnancy. Questions

1) What Is GDM? 2) How do you diagnose GDM 3) Screening test for GDM 4) When to do MOGTT 5) Name the diabetogenic hormone in

pregnancy 6) What is normal weight gain in

pregnancy? Answer

What is GDM?

A syndrome of glucose intolerance appears during pregnancy and usually disappears after pregnancy is terminated. It affects 7% of all pregnancy. It is a metabolic disorder of multiple aetiology characterized by chronic hyperglycemia with disturbances of carbohydrate, fat and protein metabolism resulting from defects in insulin secretion, insulin action, or both. Previously, it is categorized into impaired glucose tolerance test and GDM based on fasting and 2 hour post glucose load in MOGTT. However, current guidelines stated that GDM includes impaired glucose tolerance test. Diagnosing Diabetes Mellitus and the level of

blood sugar control

1) Diagnose: Based on MOGTT Normal level of MOGTT is a) Fasting: 5-6 mmol/L

b) 2 hour post glucose load: 7-8 mmol/L

2) Level of blood glucose control: Blood Sugar Profile (4-6 mmol/L) and Serum HBA1c concentration (< 6.5%)

Screening test for GDM before performing the

MOGTT

a) Random blood sugar (> 11.1 mmol/L) b) Urinary glucose level (≥ 1+ on more

than one occasion or ≥ 2+ on one occasion)

c) Mini Glucose Tolerance Test (> 7.8 mmol/L)

When to do MOGTT

1) Candidates for MOGTT is offered for this test at 16-18 weeks of pregnancy

2) If normal, then repeat at 26-28 weeks of pregnancy. If it negative, then no need to re-do it as HPL diabetogenic effect starts to plateau even though it’s serum level continue to increase proportionally.

Diabetogenic hormone in pregnancy

a) Human Chorionic Somatomammotropin

(HCS) or formerly known as Human Placental Lactogen (HPL)

b) Estrogen (stimulate production of prolactin)

c) Progesterone d) Cortisol

Notes: In GDM, besides of anti-diabetogenic hormone, there will be increased in insulin degradation by placental enzymes

Normal weight gain in pregnancy

1) First 5 months: 0.5 kg/months 2) Followed with: 0.5 kg/ week.

Page 23: Long case examination for phase iii medical students usmkk

Long Case Examination for Phase III Medical Students University Science Malaysia

16

Case: 28/M/F, G3P2 at 28W P.O.A admitted in view of uncontrolled blood sugar level. Diagnosed as GDM at 26W P.O.A. Previous pregnancy also complicated with GDM and macrosomic baby requiring LSCS. Positive family history of DM on maternal side. Questions

a) Complication of GDM b) Indication for MOGTT c) Management to this patient

Complication of GDM

Maternal a) Hypertension, ↑ incidence of pre-

eclampsia (if a/w nephropathy) b) ↑ incidence of infection – UTI,

vulvovaginitis etc c) Polyhydramnios d) Pre-term labour e) Coronary artery disease f) Thromboembolic disease g) Risk of caesarean delivery

Fetus 1. Early pregnancy a) Spontaneous abortion b) Congenital anomalies → 40% of perinatal death in diabetic pregnancies c) Cardiac defects d) Neural tube defects e) Renal anomalies f) Caudalregression synd (rare) 2. Later pregnancy a) Macrosomia b) Polyhydramnios c) IUGR (intrauterine growth restriction) d) Unexplained intrauterine death. May be secondary to: Chronic hypoxia Polycythemia Lactic acidemia Ketoacidosis

Neonate a) Congenital abnormalities b) Shoulder dystocia, birth asphyxia & traumatic birth c) Hypoglycemia – fetal islet cell hyperplasia d) Jaundice e)Respiratory distress syndrome – hyperinsulinaemia diminished surfactant production f) Hypocalcaemia and hypomagnesaemia Indication for MOGTT

1) Significant glycosuria on 2 or more occasions during pregnancy

2) Maternal obesity (i.e. maternal weight >80 kg or BMI >27 at booking)

3) Family history of diabetes in first-degree relatives

4) Previous big baby (weighing >4 kg) 5) Women >35 years old 6) Previous unexplained stillbirths,

recurrent abortions, birth defects 7) Previous history of gestational diabetes 8) Polyhydramnios in current pregnancy 9) Big baby in current pregnancy 10) Congenital abnormality

Management for this patient My point of view: This patient was diagnosed as GDM at 26W of pregnancy. Now is her 28W of pregnancy and her blood sugar level is uncontrolled. Obviously DM diet is not working. Therefore, I see the role of giving insulin injection to her. Therefore for this current admission, BSP should be done after giving insulin injection to look for the blood sugar level and further adjustment of insulin dosage. Pregnancy shouldn’t be allowed beyond 38W due to risk of unexplained IUD.

Page 24: Long case examination for phase iii medical students usmkk

Long Case Examination for Phase III Medical Students University Science Malaysia

17

36 years old Malay lady, teacher, G4P3 at 37W + 6/7 was admitted to wards for further management in view of

1) Establish DM for three years. Previously on OHA but now changed to insulin. However, blood sugar is uncontrolled. Currently there is no complication of DM develops.

2) Last pregnancy is by caesarian section because of transverse lie.

Questions

1) Option of mode of delivery and pre requisite for it.

2) Management for this patient. Answer Patient with uncontrolled diabetes mellitus should not be allowed to proceed with pregnancy beyond 38 weeks of pregnancy. Therefore, it is crucial to determine the correct date of pregnancy to avoid pre term delivery. Furthermore, fetus of diabetic mother is associated with delay lung maturity.

Mode of delivery In this patient, mode of delivery should be balanced between benefit and risk. The decision should always be discussed with the patient. Spontaneous vaginal delivery with induction of labour.

1) Should be done carefully if using Prostin because of history of c-sec with no successful VBAC. Dosage is 1.5 mg for each cycle. Membrane sweeping could be considered.

2) Need to elicit the lie of the fetus in cephalic presentation.

3) Excludes macrosomic baby.

Caesarean section 1) This is possibly a best option but this

will put patient in high risk category for next pregnancy which is 2 caesarean scars with no successful VBAC.

2) If patient wish to pregnant again, she will require caesarean section for the following pregnancy.

Management for this patient

Antenatal 1) Fetal surveillance with ultrasound for

biophysical profile and CTG. 2) Blood sugar profile with adjustment of

insulin dosage. 3) Diabetic diet

Intrapartum

1) Management based upon modes of delivery either chooses induction of labour with spontaneous vaginal delivery or caesarean section.

2) Patient should be started on DKI regimes (5% dextrose solution with 1 gram KCL) together with sliding scale insulin infusion. If patient go for c-sec, morning dose of insulin should be omitted.

3) Presence of senior obstetrician to standby in case any complication occur.

4) Pediatrician needs to be informed regarding this case.

Post partum

1) Baby should be observed in NICU for 24 hours before discharged.

2) After the delivery, insulin can be stop and patient may continue taking OHA.

3) Referral to internal medicine team for further management

4) Advise for contraception. 5) Counseling on blood sugar control if

patient wish to get pregnant again

Page 25: Long case examination for phase iii medical students usmkk

Long Case Examination for Phase III Medical Students University Science Malaysia

18

35 Years old Malay lady, G1P0 at 37W + 5/7 with gestational Diabetes Mellitus was admitted for review for intrapartum management Questions

1) What should you elicit before allowing patient to deliver by vagina delivery?

2) What is macrosomic baby? 3) Are there any differences between

macrosomic baby who is belonging to diabetic mother and non diabetic mother?

4) If this patient keen on SVD even though the estimated fetal weight is 4 Kg and the labour is complicated with shoulder dystocia, what would be your management?

Answer

Before allowing diabetic mother deliver via SVD, few thing needs to be excluded first.

1) The size of baby is not macrosomic 2) Cephalic presentation 3) Longitudinal lie 4) Not a candidate for Caesarean section

a) Major placenta previa b) Footling or flexion breech c) 2 previous c-sec scar without prior

normal delivery d) Unstable lie e) Any obstruction to descending of

fetus (fibroid, ovarian cyst, Cephalopelvic disproportion)

Macrosomic Baby

For undergraduate level, macrosomic is the estimated weight of fetus > 4 kg. However, it is further classified into categories

a) 4,000 - 4,250 g (discuss with patient regarding mode of delivery)

b) > 4,250 g = elective caesarean section Notes: Ultrasound is specific for determination of estimated fetal weight but only with sensitivity of 60-70% at term. There will be a + of 500 mg discrepancy of estimated and real fetal weight.

Macrosomic baby of diabetic vs. non diabetic

mother Macrosomic baby of non diabetic mother is at low risk for developing shoulder dystocia as compared to baby of diabetic mother. This is due to present of excessive fat tissue growth at shoulder region in baby of diabetic mother. The disproportionate excessive growth of the shoulder will predispose them to the risk of shoulder dystocia during SVD.

Steps in managing Shoulder dystocia

1) Call for help, inform senior obstetrician and pediatric colleague

2) Experienced obstetrician should be present during second stage of labour

3) Mc Roberts’ maneuvers (Flexion and abduction of the maternal hips, positioning the maternal thighs on her abdomen)

4) If not successful, apply suprapubic pressure together with Mc Roberts (External suprapubic pressure is applied in a downward and lateral direction to push the posterior aspect of the anterior shoulder towards the fetal chest )

5) If fail, proceed with Wood-Corkscrew Maneuvers (The hand is placed behind the posterior shoulder of the fetus. The shoulder is rotated progressively 180° in a corkscrew manner so that the impacted anterior shoulder is released.

6) If still fail, then deliver the posterior arm first.

7) If fail, do Zavanelli maneuvered (push the baby back) and prep for emergency C-sec

Page 26: Long case examination for phase iii medical students usmkk

Long Case Examination for Phase III Medical Students University Science Malaysia

19

Case: DM with hydrocephalus baby Question

a) H(x) and P(e) b) Investigation and management

History

1) Regarding DM a) Since when? Pre existing or during

this pregnancy b) Any history of macrosomic baby,

Polyhydramnios or unexplained IUD during previous pregnancy?

c) Are there family risk factor? d) Is MOGTT done? (normally early

pregnancy and repeated at24-28w in high risk group in which initial test is negative)

e) Now on diabetic diet, OHA, or insulin.

f) Ever being admitted due to DM complication like hypoglycaemia, diabetic foot.

g) Any complaint of DM complication like heart disease, peripheral vascular disease, diabetic nephropathy, diabetic retinopathy.

2) Regarding hydrocephalus a) How did the patient know that?

Through US (usually diagnosed after >24w)? Who confirmed it?

b) Did mother took/compliance to folic acid?

c) Did previous baby having congenital anomaly?

d) The weight of the baby? e) P(e) for unstable lie.

Investigation

1) Find the causes of hydrocephalus. TORCHES? Bleeding? Edward syndrome?

2) Observation of fetal condition through serial ultrasound. Check for any abnormality like spina bifida (associated with hydrocephalus)

3) 30 minutes CTG monitoring for fetal condition.

4) FBC and GSH for the mother 5) Blood sugar profile, Hba1c level of the

mother. 6) Check for any complication of diabetes

mellitus.

Management 1) Prenatal

a) Pre term delivery is unlikely in this case; therefore corticosteroid injection is not needed.

b) Admit the patient at obstetric wards to observe the blood sugar level. Starts with diabetic diet. If fails, starts insulin.

c) Inform the pediatrician and neonatal neurosurgeon regarding delivery of baby and next intervention. (most likely caesarian section at 38-39w to prevent head entrapment)

d) Counseling to the patient regarding the baby condition. Congenital abnormality in DM is low. On next pregnancy should take folic acid to reduce risk of hydrocephalus.

e) Termination of pregnancy is against medical ethics and Islamic law. Only fetus which is dead in vitro or no chance of living can be terminated.

2) Intrapartum a) Prep for C-sec

3) Post natal 1) Check CBS of the baby and mother 2) Admit baby to the NICU for further management.

3) Counsel mother to control diabetes and took folic acid before next pregnancy

Page 27: Long case examination for phase iii medical students usmkk

Long Case Examination for Phase III Medical Students University Science Malaysia

20

Case: Oligohydramnios

Question a) Complication of oligohydramnios b) How to detect c) Management

Definition Reduce in AFI <5 based on ultrasound [additional of vertical amniotic fluid pocket depths volume in four quadrant.] Some specialist may consider AFI <8 as oligohydramnios (AP Dr Nik Hasliza). Amniotic fluid production A) Production of amniotic fluid is from 1. Inward transfer of solute across the amnion with water following passively in early gestation. 2. Water transport across the highly permeable skin of the fetus during the first half of gestation (keratinization of skin at 22-25W) 3. Baby's urination (first starts at 8-11W and is major source of production. it is recycled when baby swallows it) 4. Secretion of large volumes of fluid each day by the fetal lungs after second half of gestation (2nd source) B) Increase amniotic fluid from 8-43W gestation linearly until 32W (700-800 mL-constant until term) -C) After 40W, declines at rate 8% per week until 300ml at 42W Causes

1) PROM or PPROM 2) fetal urinary tract anatomy (renal and

ureter most common) 3) Uteroplacental insufficiency 4) Pulmonary hypoplasia

Complication

In the term or post-term gestation, oligohydramnios is frequently associated with thick meconium (a/w Meconium Aspiration), deep decelerations in the fetal heart rate, and the dysmaturity syndrome. One team reported a 13-fold increase in perinatal mortality rate (to 57/1,000) when the sonogram showed amniotic fluid volume to be marginal, and a 47-fold increase (to 188/1,000) with severe oligohydramnios. In 62 cases of second-trimester oligohydramnios, another team reported a 43% perinatal mortality rate, with lethal pulmonary hypoplasia complicating 33% of cases. If amniotic fluid was essentially absent ("anhydramnios"), 88% had lethal outcomes, compared with 11% of those with moderate fluid reductions. Diagnosis

- Via ultrasound

Management Other Investigation

1) intrauterine instillation of dye to diagnose PROM [confirm if the dye is found in the vagina]-not practically done

2) Furosemide test to visualize fetal bladder

� Both test not practically done Others

1) Amnioinfusion of 200 ml Normal saline (not practically done)

2) Maternal rehydration.(controversial) 3) frequent fetal biophysical testing and

appropriately timed delivery 4) Rule out fetal structural and

chromosomal anomalies 5) Earlier delivery in baby incompatible

with life. Notes: risk of fetal asphyxia and death is high in IUGR

Page 28: Long case examination for phase iii medical students usmkk

Long Case Examination for Phase III Medical Students University Science Malaysia

21

35 years old Malay lady, G3P2 at 26W POA was admitted for further management after she persistently worried about her current pregnancy because her belly was too big compared to previous pregnancy Questions

1) What is polyhydramnios and how do you grade them?

2) What is the causes of polyhydramnios 3) How do you manage this patient?

Answer

Source:http://emedicine.medscape.com/article/404856-overview

Polyhydramnios

Polyhydramnios may be defined as an amniotic fluid index above the 95th centile for gestational age [Moore& Cayle]. Previously, it is defined when the deepest vertical pool is more than 8 cm, but currently based on measurement on 4 quadrant > 25. (Based on ultrasound) It complicates approximately 0.4-3.5 % of pregnancies and it can be divided into three groups: mild (amniotic fluid index 25-30),

moderate (AFI 30.1-35) and severe (AFI >35) [Naser Omar et al]

Causes of polyhydramnios

1) 60% is idiopathic 2) Maternal causes

a) Gestational diabetes mellitus 3) placental abnormalities (placental

abruption, placenta accreta) 4) Fetal factor

a) congenital anomalies ( anencephaly, hydrocephalus, spina bifida, tracheoesophageal fistula, duodenal atresia, hydrops fetalis and many more)

b) Multiple pregnancy c) chromosomal abnormalities such as

Down's syndrome and Edwards syndrome

5) Skeletal dysplasia and syndrome. 6) others like chorioangioma of the

placenta

Management to this patient

1) Reassure the mother 2) Excludes the causes of polyhydramnios

a) This patient should be offered to do MOGTT

b) Ultrasound examination and proceed to Doppler and full scan if necessary

3) Assessment of fetal well being a) Access while doing ultrasound +

CTG. 4) Treat the underlying causes 5) Treat the hydramnios

a) Mild & Moderate: Indomethacin or sulindac

b) Severe: Amnioreduction 6) Corticosteroid if anticipating pre term

delivery.

95th

percentile

5th

percentile

Mean value

Page 29: Long case examination for phase iii medical students usmkk

Long Case Examination for Phase III Medical Students University Science Malaysia

22

Case: 32/M/F, G2P1, decreased fetal movement Question

1) H(x) and P(e) 2) Management of decrease fetal

movement 3) Use of tocolytic (function/type) 4) Fetal kick chart (indication and

component) Reduce fetal movement? Baby goes through normal sleep cycle. As long as baby moves every couple of hour, then it’s fine.

History Exclude Abruptio placenta -Decreased fetal movement, abdominal pain, bleeding after 22w -shocks, tender uterus, fetal distress/absent fetal heart sound Exclude fetal distress -Decreased/absent fetal movement, abnormal fetal heart rate - Thick meconium stained fluid Other history - What did patient do? Working mother seems to perceive less fetal movement. - Any history of trauma? - Elicit maternal medical illness PE and investigation

1) Auscultation of fetal heart rate and confirmation with ultrasound.

2) CTG monitoring for ½ hour. 3) Umbilical artery Doppler ultrasound in

high risk cases.

Tocolysis The administration of medications to stop uterine contractions during premature labor

Tocolysis has also been advocated for the management of intrapartum fetal distress, impaired fetal growth, pre term labour and to facilitate external cephalic version at term MgSO4

1. works as membrane stabilizer, competitive inhibition of Ca; therapeutic at 4-7 mEq/L

2. SE: flushing, nausea, lethargy, pulm edema

3. Toxicity: cardiac arrest (tx: calcium gluconate), slurred speech, loss of patellar reflex (@ 7 -10), resp problems (@15-17), flushed/warm (@9-12), muscle paralysis (@15-17), hypotonia (@10-12)

Nifedipine

1) calcium channel blocker: 10 mg q 6 h; se: nausea and flushing

B2 agonist

1. ritodrine/ terbutaline 2. dec. uterine stimulation; may cause

DKA in hyperglycemia, pulm edema, n/v, palpitations (avoid with h/o cardiac disease or if vaginal bleeding) 0.25 mg sq q 20-30 min x 3 then 5 mg q 4 po

Indomethacin/prostaglandin synthesis inhibitor

1. 50 mg po/100 mg pr SE: premature closure of PDA in an hour,oligohydramnios

Fetal kick chart

1) Screening by caregivers to alert them about their fetal condition which might compromised. This will aid early intervention to reduce perinatal mortality. 2) Routine or done in women with increased risk of complication in baby 3) Decision of management shouldn’t be made based on fetal kick chart.

Page 30: Long case examination for phase iii medical students usmkk

Long Case Examination for Phase III Medical Students University Science Malaysia

23

Case: Reduce fetal movement Question a) Regarding traditional medicine, how to

advice patient b) Line of thinking to get diagnosis c) Management.

Traditional medicine

A doctor has no right to order patient to stop taking traditional medicine. However, lack of study and information between interaction of traditional medicine and modern medicine may cause few un-expected side effect. Furthermore, few manufacturers being dishonest by adding some ‘hidden’ ingredient inside their product which may cause serious side effect in reaction to certain drugs. Therefore, as a doctor we can advise patient to

1. Choose either taking only traditional or modern medicine or not combining them.

2. Suggest to them to stop traditional medicine while pregnant because afraid of unexpected side effect with prescribed medicine.

3. Avoid herbal base traditional medicine. 4. Use alternative traditional medicine that

known scientifically not harmful like honey.

Line of thinking to get the diagnosis

1) Is mother really paying full attention about fetal movement a) Fetal movement is rather perception

of woman. Busy mother tends to feel less fetal movement.

b) Working in busy environment may cause less perception of fetal movement.

c) A woman which is first time pregnant may become too anxious about fetal condition and notice

about decrease in fetal movement as compared with multi para.

2) Identification of maternal risk factor which might contribute to perinatal mortality.

- age, smoking, overweight/obesity, previous stillbirth or neonatal death

3) What actually the causes of reduce or

absent fetal movement? a) Placenta Abruptio b) Intra uterine growth restriction c) Syndromic baby d) Placenta insufficiency. e) Mother’s perception.

4) Investigation to support diagnosis

Management 1) Take full history and elicit risk factor

that might compromise fetal condition. 2) Fetal well being assessment

(recommended by NICE guideline) CTG, Ultrasound.

3) Fetal kick chart (not recommended by NICE and others as it will cause more anxiety to the mother.) however, some says it is better than doing nothing.

4) If CTG or ultrasound shows fetal compromise, admit patient to the wards and do serial monitoring of fetal condition

5) Re assures the mother. 6) Patient can be safely discharge after

fetal monitoring shows normal result in three consecutive days. Discharge patient with a) TCA at antenatal wards weekly or

twice weekly b) To come again to ward if reduce

fetal movement c) Instruction to use fetal kick chart.

Page 31: Long case examination for phase iii medical students usmkk

Long Case Examination for Phase III Medical Students University Science Malaysia

24

Case: Premature labour with PV Bleeding Questions

a) 4 drugs in management of premature labour

b) Drugs for candidiasis c) Function, complication and monitoring

of the drugs d) Doses of drugs

Definition of preterm

1) Onset of labour after the gestation of viability i.e 24 weeks and before 37 completed weeks of pregnancy. 2) The onset labour may be determined by documented uterine contractions and rupture membranes or documented cervical change with an estimated length of less than 1 cm and/or cervical dilatation of more than 2 cm.

Types a) Threatened (uterine contraction without cervical changes) b) Actual/establish (uterine contraction+ cervical changes) Additional: occurs in around 7% of all pregnancies and is a major cause of infant mortality and morbidity. [Scottish guidelines] Survival rate: 23 w 0-8% 24w 15-20% 25w 50-60% 26-28w 85% 29w 90% Drugs in management of premature labour

1) Corticosteroid therapy - Betamethasone, 12mg, IM, 24 hours

apart. - In USM, Dexamethasone, 12 MG, IM,

12 hours apart. - Function is to increase lung maturity.

Usage of corticosteroid below 24w is no beneficial since pneumocyte not develop yet. Also not recommended >34W.

- Possible long-term effects on cognitive or neurological development, impaired

glucose tolerance, osteoporosis and depression of fetal/maternal adrenals

2) Tocolytic Nifedipine - calcium channel blocker: 10 mg qds; - se: nausea and flushing B2 agonist - ritodrine/ terbutaline - dec. uterine stimulation; may cause

DKA in hyperglycemia, pulm edema, n/v, palpitations (avoid with h/o cardiac disease or if vaginal bleeding) 0.25 mg sq q 20-30 min x 3 then 5 mg q 4 po

3) Antibiotic therapy - For women at risk of preterm delivery

because of PPROM, prophylactic antibiotics delay delivery and reduce maternal and neonatal infective morbidity.

- Not recommended in risk of preterm but with intact membranes

- Erythromycin 500mg qds plus co-amoxyclav (Augmentin) 375mg tds for 7 days OR clindamycin 150mg qds for 7 days.

Drug for candidiasis in pregnancy

Imidazoles are best but pregnant women may need longer (7 not 4 day) courses. Thrush is a common vaginal infection in pregnancy causing itching and soreness. There is no evidence that this yeast infection harms the baby. Antifungal creams are effective. Imidazoles (such as clotrimazole) are more effective than older treatments such as nystatin and hydrargaphen. Longer courses (7 days) cured more than 90% of women whereas standard (4 day) courses only cured about half the cases. [Cochrane Database of Systematic Reviews, Issue 4, 2009]

Page 32: Long case examination for phase iii medical students usmkk

Long Case Examination for Phase III Medical Students University Science Malaysia

25

Case: 33 years old, G3P2 present at 24W+5D POA with premature contraction. No history of UTI, Vaginal discharge, trauma. Question

A) How to differentiate premature contraction and true labour contraction

B) Management of this patient C) How to discharge this patient

Premature contraction

Uterine contraction after the gestation of viability. i.e 24W and before 37 completed weeks of pregnancy. It could progress to premature labour. It is called threatened pre term labour if contraction is not associated with cervical dilatation. If it is associated with cervical dilatation, hence it is termed as Establish Pre term labour.

Characteristic of a true labour 1) At term 2) Come at regular interval i.e once in one

hour and finally can goes to once in five minutes near labour.

3) The timing of each contraction is last about 30-70 seconds

4) The intensity of pain increase by time. Pain is at the back due to referred pain of cervix.

5) The pain does not relief by walking or changing in posture.

6) Presence of show and liquor.

Management of this patient 1) Obtain full history and perform relevant physical examination. 2) Admit patient to premature room in labour room. 3) Inform the case to MO in charged

4) Re assure the patient 5) Give IM Dexamethasone 12mg bds, 12 hours apart. 6) Keep patient nil by mouth and anticipate for caesarian section. 7) Hydrate patient adequately with 2 pints NS and 3 pints D5% 8) Take blood for investigation including FBC, GSH. 9) Urine dipstick (nitrogen, albumin= indicate UTI) and Urine FEME. 10) Allocate possible causes of premature contraction. 11) Monitor 4 hourly BP, 20 minutes CTG 12) Inform Pediatrician regarding patient's condition and keep in view to book for ventilator. 13) Monitor the labour progression by labour progression chart. 14) Ultrasound examination for fetal well being. 15) Administer tocolytic for example Nifedipine 5mg (some specialist give 10 mg) 16) Observe patient for one day. if contraction subside, discharge patient to ante natal wards for further observation. 17) If contraction subsides for two consecutive days in ante natal wards, then patient can be discharged

Discharging the patient 1) After no contraction within 2 days in ante natal wards 2) Ensure that patient already took dexamethasone. 3) CTG reactive. 4) Follow up at ante natal clinic within 2 weeks. 5) Fetal Kick Chart (some protocol say it is not indicated)

Page 33: Long case examination for phase iii medical students usmkk

Long Case Examination for Phase III Medical Students University Science Malaysia

26

Case: 42/M/F G11P7+3A at 36/52 with history of 5 premature deliveries, electively admitted for removal of cervical cerclage Questions

a) Risk of grandmultiparae b) Risk of pregnancy at old age c) How to prevent PPH in this patient d) Indications of cervical cerclage e) When to do and remove cervical

cerclage.

Grandmultiparae Definition: a woman who has had five or more pregnancies resulting in viable fetuses. Great grand multipara if > 10 The results showed high in incidence of anemia (80%). Cesarean section (38% vs 35%), inversion of uterus (0.2% vs nil) and rupture of uterus (0.2% vs nil), hypertension and PIH superimposed on chronic hypertension (12.5-25% vs 8-14%). The incidence of postpartum hemorrhage, abruptio placentae, preterm labour, obstructed labour, puerperal sepsis and wound infection was also high in grand multi parous group. There were 9 maternal deaths out of 1000 cases of study group as compared to 4 deaths in control group. Similarly the perinatal mortality rate was 180/1000 births as compared to 150/1000 in para 2-4. [Grand Multiparity: Still an Obstetric Risk Factor,Khadija H Asaf. Pak J Obstet Gynaecol May 1997;10(1,2):24-8]

Pregnancy at old age an age over 35 years for the ‘elderly primigravida’(FIGO, 1958)Improvements in women’s general health have led to this term tending to be reserved for pregnancies in women at or above 40 years of age.[Current Obstetrics & Gynaecology (2005) 15, 46–53] Risk of pregnancy at old age: Miscarriage, ectopic pregnancy, chromosomal disorder in

fetal, placenta abnormalities (PE, Anruptio placenta, PP), multiple pregnancy, Medical condition like Hypertension, DM and anemia, aneuploidy and fetal anomalies, increase maternal mortality. [The older obstetric patient, Current Obstetrics & Gynecology (2005) 15, 46–53]

Cervical cerclage Cervical cerclage is a procedure in which sutures are inserted around the cervix in women suspected to have cervical weakness. This is thought to prevent cervical dilatation and membrane exposure, thus helping the uterus to retain the pregnancy in women who are prone to miscarrying, mostly in the mid-trimester. [Cervical cerclage, Current Obstetrics & Gynaecology (2006) 16, 306–308] Cervical cerclage can be classified as an elective procedure (based on previous history and/or investigation), a selective procedure (based on evidence obtained by ultrasound examination that shows shortening of the cervix) or an emergency procedure (when the cervix is dilated with the membranes seen or bulging via the cervical os). An elective procedure is performed around 12–14 weeks’ gestation (Dr Amir HUSM-16W) after confirming fetal viability. The TAC is performed around the same time. Emergency cerclage is performed when the cervix is noted to be dilating.[Cervical cerclage, Current Obstetrics & Gynaecology (2006) 16, 306–308] It is removed when term is achieved or premature contraction (Dr Amir HUSM)

Page 34: Long case examination for phase iii medical students usmkk

Long Case Examination for Phase III Medical Students University Science Malaysia

27

Case: Post EDD + PIH (exact case is unknown so discussion is random)

Question

a) Differential diagnosis b) Investigation c) Management

Term Period of gestation 37 to 42 week Post date Post date is a term to describe any pregnancy that goes beyond expected date of delivery (40 W) but does not exceed term. Current practice in HUSM and KKM is to avoid the delivery of post term baby due to the increase perinatal morbidity and mortality associated with post term. Therefore, induction of labour should be initiated if pregnancy goes beyond the post date. Approach to this patient

1) Absolute treatment for PIH is the termination of pregnancy.

2) Post EDD itself is an indication for induction of labour to prevent post term delivery.

3) However, the exact POG needs to be established to avoid delivery of pre term baby.

Investigation

For Post EDD 1) Biophysical profile of the baby

• Fetal tone • Movement of the body or limbs • breathing movement • Amniotic fluid volume • Heart rate (CTG) analysis.

2) Doppler ultrasound

For PIH 1) For baby(similar to post EDD) 2) For mother

a) Hourly BP monitoring b) PE Profile (platelet count, uric acid,

serum creatinine level, Liver enzymes- AST, Urine albumin)

Management for Post date and post Term

Based on scientific evidence 1) Women with post term gestations who

have unfavorable cervices can either undergo labor induction or be managed expectantly.

2) Prostaglandin can be used in post term pregnancies to promote cervical ripening and induce labor.

3) Delivery should be effected if there is evidence of fetal compromise or oligohydramnios.

Based on expert experiences

1) Antenatal surveillance for post term pregnancies between 41 weeks (287 days; EDD +7 days) and 42 weeks (294 days; EDD +14 days) of gestation because of evidence that perinatal morbidity and mortality increase as gestational age advances.

1. Twice-weekly testing with some

evaluation of amniotic fluid volume beginning at 41 weeks of gestation. A nonstress test and amniotic fluid volume assessment (a modified BPP) should be adequate.

Drug commonly use in PIH a) Methyl dopa b) Labetolol c) Nifedipine d) Magnesium Sulphate (pre eclampsia)

Page 35: Long case examination for phase iii medical students usmkk

Long Case Examination for Phase III Medical Students University Science Malaysia

28

31 years old Malay lady, G6P5 at date + 9/7 with 1 previous scar for transverse lie and 4 VBAC

1) What is post term? 2) What is the complication of the post

term? 3) In this patient, how will you manage her

and give reasons.

Post term A pregnancy that has extended to or beyond 42 weeks of gestation (294 days, or estimated date of delivery [EDD] +14 days) [ACOG guidelines]

Complication of the post term

1) To the baby a) Uteroplacental insufficiency b) Oligohydramnios causing cord

compression syndrome c) Meconium aspiration syndrome d) Intrauterine infection e) Macrosomia and complication related to

it f) Fetal dysmaturity syndrome g) Increased risk for neonatal

encephalopathy

2) To mother a) Severe perineal injury if delivering big

baby b) Increase rate for caesarian section c) endometritis, thromboembolic disease,

hemorrhage. d) Psychological (anxiety and frustration)

for carrying the baby longer than expected.

Management for this patient

Women with an uncomplicated pregnancy who reach 41 to 42 weeks’ gestation should be offered elective delivery [SOGC]

In this patient, we however need to go through a broad perspective before deciding the management for this patient. Patient’s problem is

a) Post date b) 1 previous c-sec with successful 4

VBAC (low risk patient) c) Grand multi para (deliver > 5 times)

Therefore, risk and benefit on method of delivery should be considered. Roughly, this is the overview of the management.

1) Antenatal surveillance at 42 to 42 week including at least non stress test and assessment of amniotic fluid volume.

2) Estimating the fetal weight. 3) Considering the induction of labour

a) Prostin and oxytocin is not a good choice.

b) May consider membrane sweeping c) After excludes macrosomic baby,

breech presentation and severe oligohydramnios.

4) If we considering Caesarian section a) Indicated if it is a macrosomic baby b) But this will put mother on higher

risk on next pregnancy because of two c-sec scar and grand Multipara. Risk of uterine atony and rupture is high

c) Advise on bi tubal ligation for the mother.

d) Indicated if breech presentation. e) Indicated in severe oligohydramnios

5) Present of senior MO and pediatrician during delivery.

Page 36: Long case examination for phase iii medical students usmkk

Long Case Examination for Phase III Medical Students University Science Malaysia

29

A healthy 25-year-old nulliparous woman has an uncomplicated pregnancy at 42 weeks. Induction of labour is fully discussed, suggested and declined. Evaluate the tests that may be arranged to monitor fetal health until labour begins. [Journal review] Where the advice over induction is unacceptable to the patient, various measures may be used to review fetal health. Several tests are described for this situation, but the problem remains that no test or group of tests has been shown to improve the perinatal outcome. Fetal movement charts have long been used in late pregnancy as a form of fetal monitoring. They require the woman to note the time taken for 10 fetal movements. A large randomised-controlled trial (RCT), however, demonstrated no reduction in fetal mortality compared with the control group, and the use of these charts may increase anxiety without a beneficial effect. Various forms of fetal acoustic stimulation test have been developed using a transabdominal sound source and assessing the fetal reaction in terms of cardiotocograph (CTG) changes and fetal movements. Although such tests have the advantage of taking less time than other tests, there is no evidence for an improved perinatal outcome. The standard non-stress test (NST) using a CTG relies on the observation of two or more episodes of fetal cardio acceleration of 15 beats or more occurring within 20 minutes of the onset of the test. It is generally thought, and advised by the National Institute for Health and Clinical Excellence (NICE), that twiceweekly tests should be performed, although the optimum scheme is not known. Furthermore, the NST used alone has a low sensitivity. The most common cause of perinatal death in such cases is meconium aspiration due to an acute asphyxial event, and the NST is not adequate to preclude

such events. Contraction stress tests assessing fetal heart responses to oxytocic-induced contractions have largely gone out of use as the high false-positive rate has led to a high level of intervention. They also take longer and are more complicated to conduct than NSTs. The assessment of liquor volume that may be related to placental function and fetal health has become an accepted part of surveillance of women such as this. A measurement of the amniotic fluid index of less than 5 cm or of the maximum vertical pocket depth of less than 2 cm (various other levels are quoted) suggest fetal compromise and lead to a recommendation for delivery. The biophysical profile combines an ultrasound assessment of fetal movements and tone, breathing movements and amniotic fluid volume. This combination would seem to be the most appropriate, but a recent RCT showed no advantage over CTG with amniotic depth measurement. Doppler studies of umbilical artery velocimetry have not been shown to be of benefit in predicting outcome. There are clearly considerable limitations in the value of all these tests in detecting fetal compromise and enabling rescue, but NICE recommends twice-weekly CTG and measurement of amniotic pool depth, and this will remain the advice until further advice based on RCTs is available. The advice and its reasons must be discussed with this woman, and a sensitive approach is most likely to lead to a compromise, although there remains a possibility that the woman will wish for no tests and will await nature’s decision. Simon G. Crocker Department of Obstetrics & Gynaecology Norfolk & Norwich University Hospital, Colney Lane, Norwich NR4 7UY, UK [OBSTETRICS, GYNAECOLOGY AND REPRODUCTIVE MEDICINE 17:1,2007 Published by Elsevier Ltd.]

Page 37: Long case examination for phase iii medical students usmkk

Long Case Examination for Phase III Medical Students University Science Malaysia

30

Case: PPROM Question:

a) Symptoms of fever b) Positive findings in PPROM c) Ix and Mx d) Causes of unstable lie

PPROM Membrane rupture that occurs before 37 weeks of gestation is referred to as preterm PROM Intraamniotic infection has been shown to be commonly associated with preterm PROM, especially if preterm PROM occurs at earlier gestational ages. In addition, factors such as low socioeconomic status, second- and third-trimester bleeding, low body mass index less than 19.8, nutritional deficiencies of copper and ascorbic acid, connective tissue disorders (eg, Ehlers–Danlos syndrome), maternal cigarette smoking, cervical conization or cerclage, pulmonary disease in pregnancy, uterine overdistention, and amniocentesis have been linked to the occurrence of preterm PROM The risk of recurrence for preterm PROM is between 16% and 32%. Fever Fever is considered as temp above 100.40 (380C) but feverish sensation may occur when body temp above 98.60 (370C)

Symptoms 1) Patient complaints of body become hot

and sweating (increase temperature and diaphoresis)

2) Can also a/w with tachycardia, altered consciousness, chills & rigor, headache, muscle and joint pain.

Positive findings in PPROM

1) Clear watery and alkaline per vaginal discharge. (pH 7.1-7.3 compared with vaginal pH 4.5-6.0)

2) Arborization (ferning) under microscopic visualization

3) Oligohydramnios [ACOG Practice Bulletin, VOL. 109, NO. 4, APRIL 2007] Symptoms of chorioamnionitis: High grade fever, maternal and fetal tachycardia, tender uterus.

Investigation and management **Based on period of gestation but basically

1. Determination of gestational age, fetal

presentation, and well-being 2. Expeditious delivery in patient with

evident intrauterine infection, abruptio placenta, or evidence of fetal compromise

3. swabs for diagnosis of Chlamydia trachomatis and Neisseria gonorrhoeae if immediate delivery not indicated

4. Group B antibiotic prophylaxis 5. CTG monitoring for umbilical cord

compression or asymptomatic uterine contraction.

Causes of unstable lie

2. Prevention of head descending a) Cephalopelvic disproportion b) Fibroid c) Ovarian cyst d) Placenta previa e) Uterine surgery f) Multiple gestation g) Fetal abnormality (anencephaly) h) Fetal neuromuscular disorder

3. Condition that permit free fetal

movement a) Polyhydramnios (AFI>8) b) Uterine laxation

Page 38: Long case examination for phase iii medical students usmkk

Long Case Examination for Phase III Medical Students University Science Malaysia

31

25 years old Malay lady, G3P2 at 40 weeks + 2/7 of pregnancy presented to you because of gushing of clear fluid from vagina for 1 day duration. However, there is no contraction pain Questions

1) Differential diagnosis and further history to support your diagnosis

2) What complication that should concern you that may occur to the mother and baby.

3) How do you manage this patient

Differential diagnosis 1) Pre labour rupture of membrane - Establish that the fluid is truly amniotic

fluid and not urine. - Contraction pain - Liquor color

2) In labour - Contraction pain that become shorten in

intervals

3) Bacterial vaginosis - Foul smelling discharge - Itchiness of vagina - Yellow or cream color discharge - Any systemic sign for infection.

Complication to look for

1) Chorioamnionitis Notes: chorioamnionitis must be anticipated in patient presented with PPROM or PROM as it can cause death to the mother and fetus. Beware sign and symptoms of chorioamnionitis a) Fever b) Maternal tachycardia c) Tender uterus d) Fetal tachycardia

Management 1) Close observation of vital sign 2) Review patient regularly especially

palpation of uterus. a) Elicit uterine tenderness b) Lie and presentation of the fetus c) Estimated fetal weight.

3) Assessment of fetal well being (CTG and ultrasound )

4) Rehydrate the patient 5) IV ampicillin as prophylaxis against

GBS (rate of infection rise after 12 hour rupture of membrane)

6) Corticosteroid is not indicated. 7) Notify the pediatric team regarding the

possibility to admit the baby because of infection.

8) This patient should not be discharged and induction of labour should be discussed with patient if not deliver after >24 hour. Usually 90% of patient with PROM will deliver within 24 hour.

Notes: Student must be able to differentiate between the PPROM and PROM. PPROM is premature rupture of membrane. i.e; membrane rupture during the period of fetal viability (>24 W) but not reach term yet. Meanwhile, Prelabour rupture of membrane, PROM is the ruptured membrane before labour. The patient already at term. In PPROM, the focus is 1) to prolong the pregnancy so that the chances for fetus to survive remain high and 2) to prepare the baby for possibility to be delivered prematurely. In PROM, a focus should be stress on possible infection to the mother and also to the fetus. Lung complication to the fetus is unlikely as lung maturation already completed.

Page 39: Long case examination for phase iii medical students usmkk

Long Case ExaminationUniversity Science Malaysia

Case: Placenta Previa Question

a) Management b) Other name for low transverse scarc) Types of placenta previa d) Investigation to order e) Complication f) How to differentiate between placenta

previa and abruptio. Definition: Placenta implanted in the lower segment of the uterus, presenting ahead of the leading pole of the fetus. It occurs in 2.8/1000 singletonpregnancies and 3.9/1000 twin [MARCH JOGC MARS 2007]

Grade.

I Placenta encroaches on the lower uterine segment but does not reach the cervical os.

Lateral

II Placenta reaches the margin of the cervical os but does not cover it.

marginal

III Placenta partially covers the os.

IV Placenta is symmetrically implanted in the lower uterine segment

Complete

Other name for low transverse scarPfannenstiel (traditional) scar, Joel-

Long Case Examination for Phase III Medical StudentsUniversity Science Malaysia

32

Other name for low transverse scar

How to differentiate between placenta

lower segment of the leading pole of

the fetus. It occurs in 2.8/1000 singleton pregnancies and 3.9/1000 twin pregnancies

Minor

major if posterior located

Major

Other name for low transverse scar -Cohen scar.

Investigation to order1) Transvaginal or Abdominal US2) FBC, GSH, Rh compatibility3) CTG

Management1) Admission to ward for PP Major.2) Bed rest and serial US as placenta

migration can occur. 3) Transfuse blood if needed i.e

symptomatic or near delivery, (target Hb at delivery is at least 8) + haematinic.

4) Tocolysis and corticosteroid if prem delivery is anticipated

5) Avoid Sexual intercourse6) PP Minor can be allowed for SVD7) Counseling to the patient.

Complication:Antepartum -APH (3rd trimester) and Cx of blood transfusion-Unstable lie - Perinatal death - Pre term labour -Thromboembolism d/t prolongs bed rest.- Placenta abruption Intrapartum - Excessive bleeding during SVD- C- section and its complication (major PP)- Hysterectomy Post partum -DIC - Intra uterine adhesion -Recurrent PP - Placenta accrete

PP VS Placenta abruptioAssociation with pain: PP is painlessUS: abruptio shows Retro placental blood clot

for Phase III Medical Students

Investigation to order Transvaginal or Abdominal US FBC, GSH, Rh compatibility

Management Admission to ward for PP Major. Bed rest and serial US as placenta

Transfuse blood if needed i.e symptomatic or near delivery, (target Hb at delivery is at least 8) + haematinic. Tocolysis and corticosteroid if prem

Avoid Sexual intercourse PP Minor can be allowed for SVD Counseling to the patient.

Complication:

trimester) and Cx of blood transfusion

Thromboembolism d/t prolongs bed rest.

Excessive bleeding during SVD section and its complication (major PP)

PP VS Placenta abruptio painless

shows Retro placental blood clot

Page 40: Long case examination for phase iii medical students usmkk

Long Case Examination for Phase III Medical Students University Science Malaysia

33

21 Years Old Malay Lady, G1P0 at 36/52 + 4/7 was admitted since 30 Week POA after history of sexual intercourse and strenuous activity. Currently, there are no more bleedings. Ultrasound reveals placenta previa type III Questions

1) How to differentiate PP type III and PP type IV on ultrasound

2) What is the risk factor for placenta previa

3) Can this patient be discharged? 4) Management for this patient

How to differentiate PP type III and PP type IV

on ultrasound PP type III- Placenta partially covers the os. PP type IV- Placenta is symmetrically implanted in the lower uterine segment

Risk factor for placenta previa - Multiple gestation - Previous caesarian section - Uterine structure anomaly - Assisted conception - Dilation & Curettage

Can this patient be discharged?

No, Women with major placenta praevia who have previously bled should be admitted and managed as in patients from 34 weeks of gestation. [RCOG] All women at risk of major ante partum hemorrhage should be encouraged to remain close to the hospital of confinement for the duration of the third trimester of pregnancy [Royal Australian and New Zealand College of Obstetricians and Gynecologists] Women with placenta praevia who have bled tend to deliver earlier

Management for this patient 1) FBC to access the level of hemoglobin

of this patient 2) GSH as patient may lose lot of bloods

during delivery. GXM 2 unit should be prepared before delivery.

3) Tocolysis as bleeding in PP patient may also due to contraction of uterus (not the lower segment)

4) Assessment of fetal well being 5) Complete bed rest and avoid any

excessive activity. 6) Discuss with patient regarding caesarian

section as mode of delivery. Prior to delivery, all women with placenta praevia and their partners should have had antenatal discussions regarding delivery, hemorrhage, possible blood transfusion and major surgical interventions, such as hysterectomy, and any objections or queries dealt with effectively.[RCOG]

7) To prepare the patient for caesarian section.

Notes:

1) Trans vaginal ultrasound is safe in the presence of placenta praevia and is more accurate than trans abdominal ultrasound in locating the placenta [RCOG Guideline No. 27

2) Placenta migration occurs during second and third trimester except in posteriorly located PP and present of C-sec scar.

3) A scan should be performed at 32 weeks in case suspected PP major and 90% of the patient who is diagnosed with PP major will remains so.

4) Elective caesarean section should be deferred to 38 weeks to minimize neonatal morbidity.

Page 41: Long case examination for phase iii medical students usmkk

Long Case Examination for Phase III Medical Students University Science Malaysia

34

Case: 42/M/F G9P8 with unstable lie

a) Causes of unstable lie b) Management c) Complication

Unstable lie 1. Fetal lie and presentation repeatedly

change at beyond 36/52 of gestation. 2. by 36W, fetal movement is limited, fetal

should present as cephalic) 3. Incident at 26/32 is 40%, at 30/52 is

20% & at term is 3%

Causes of unstable lie 4. Prevention of head descending

i) Cephalopelvic disproportion j) Fibroid k) Ovarian cyst l) Placenta previa m) Uterine surgery n) Multiple gestation o) Fetal abnormality (anencephaly) p) Fetal neuromuscular disorder

5. Condition that permit free fetal

movement c) Polyhydramnios (AFI>8) d) Uterine laxation

History

a) Make sure that the date is correct cause unstable lie is physiological <36/52.

b) Find any risk factor associated with unstable lie.

c) Elicit any problem during pregnancy

Management

1) Admit patient to antenatal wards a) Daily observation for fetal lie b) Provide active management to

correct lie c) Provide immediate clinical

assistance upon membrane rupture 2) Exclude factors contributing to unstable

lie 3) Expectant vs. Emergent management

Expectant A) Daily observation for fetal lie B) Discharge if longitudinal lie for 3

consecutive days C) Review patient in a week time D) Wait for spontaneous labour

Active management

A) Caeserean section B) ECV C) Stabilizing induction of labour

Complication

1) Cord prolapsed leading to fetal hypoxia/ fetal death.

2) Compound presentation 3) Uterine rupture

Page 42: Long case examination for phase iii medical students usmkk

Long Case Examination for Phase III Medical Students University Science Malaysia

35

Case: Unstable lie Question:

a) Physical examination (abdomen) b) Level of exposure c) Clinical evidence of head and buttock d) Management for this patient e) Option for this patient f) What is unstable lie g) Causes of unstable lie

Unstable lie

1. Fetal lie and presentation repeatedly change at beyond 36/52 of gestation.

2. by 36W, fetal movement is limited, fetal should present as cephalic)

3. Incident at 26/32 is 40%, at 30/52 is 20% & at term is 3%

Clinical evidence of head and buttock Head: Hard, round and ballotable Buttock: Soft, broad and not ballotable.

Management

1) Admit patient to antenatal wards a) Daily observation for fetal lie b) Provide active management to

correct lie c) Provide immediate clinical

assistance upon membrane rupture 2) Exclude factors contributing to unstable

lie 3) Expectant vs Emergent management

Expectant A) Daily observation for fetal lie B) Discharge if longitudinal lie for 3

days C) Review patient in a week time D) Wait for spontaneous labour

Option for this patient

A) Passive management by observation in hope that the lie will return to normal position during term.

B) Caeserean section C) ECV [Relative contraindication]

Results vary from 30% up to 80% in different series. Race, parity, uterine tone, liquor volume, engagement of the breech and whether the head is palpable, and the use of tocolysis, all affect the success rate. [Greentop]

D) Stabilizing induction of labour

Notes: B-D is active management.

Complication

1) Cord prolapsed leading to fetal hypoxia/ fetal death.

2) Compound presentation 3) Uterine rupture

Causes of unstable lie

Prevention of head descending a) Cephalopelvic disproportion b) Fibroid c) Ovarian cyst d) Placenta previa e) Uterine surgery f) Multiple gestation g) Fetal abnormality (anencephaly) h) Fetal neuromuscular disorder

Condition that permit free fetal movement a) Polyhydramnios (AFI>8) b) Uterine laxation

Page 43: Long case examination for phase iii medical students usmkk

Long Case Examination for Phase III Medical Students University Science Malaysia

36

Case: Breech Questions

a) Causes and complication of breech b) Management, mode of delivery and time

of delivery for breech. c) ATT- Type of immune

Breech It is the most common type of malpresentation. Presentation of the fetal buttocks and feet in labour Incidence: 26W 40%, 30W 20%, term 3% Type: Extended, Flexed, Footling

Causes 1) Multiparous woman with lax uterus and

abdomen 2) Prematurity 3) Fetal structural anomalies; anencephaly,

hydrocephalus 4) Uterine anomalies; uterus bicornu,

fibroids 5) Multiple gestation; twins 6) Hydramnios; oligo or poly 7) Placenta previa 8) Contracted maternal pelvis 9) Pelvic tumours

Complication

1) PROM 2) Cord prolapsed [common in footling

presentation and lesser in flexed breech presentation]

3) Difficulty in delivering the shoulder 4) Difficulty in delivering the head[ may

lead to intracranial bleeding d/t tear of tentorium or delay delivery of head can cause prolonged compression of cord and asphyxia]

5) Birth trauma such as fracture, viscera damage, Erb Duchenne paralysis, dislocation of hip joint.

Mode and timing of delivery

<28 weeks, weight <1 kg

SVD

28-32 weeks, weight 1.0 - 1.5 kg

LSCS

32-37 weeks Weight 1.5 – 2.5 kg

Depend on case 1) Assisted

breech delivery for Extended and flexed

2) LSCS for footling breech

> 37 weeks

Preferably Caesarean section

ATT

Tetanus vaccine is an inactivated toxin (toxoid) made by growing the bacteria in a liquid medium and purifying and inactivating the toxin. Type II Immune response It is administer once the quickening felt and can be repeated 2-3 months after first injection in primid women as a booster injection. (Usually 5th and 7th months of pregnancy) Notes: It is different from Anti tetanus human immunoglobulin which are preparation containing IgG immunoglobulin derived from plasma of donors sensitized to tetanus toxoid. It acts by The IgG antibodies acts to neutralize the free circulating exotoxisn of clostridium tetani and prevent its fixation in tissue and its consequences.

Page 44: Long case examination for phase iii medical students usmkk

Long Case Examination for Phase III Medical Students University Science Malaysia

37

35 years old Malay lady, G5P5 (1 pair twin) at 26/52 of pregnancy was admitted to wards because of premature contraction and twin pregnancy. Questions

1) What history that could give you idea that you are dealing with cases of multiple pregnancy?

2) How do you diagnosed multiple pregnancy through physical examination

3) How do you classify multiple pregnancy 4) Complication of multiple pregnancy

Answer Notes: Account for only 3% of all live births, they responsible of a disproportionate share of perinatal morbidity and mortality History

- Accelerated weight gain - Hyperemesis gravidarum - Sensation of moving of more than one

fetus - Infertility treatment by ovulation-

inducing agents or gamete/zygote transfer

- family history of dizygotic twins Notes: certain race like Africa has higher risk factor to have multiple pregnancies.

Physical examination

- Presence of more than 2 poles (need to excludes fibroid)

- Usually, the abdomen size is bigger than corresponds date

- Polyhydramnios - Presence of two or more fetal heart

sounds on pinnard auscultation. Classification of multiple pregnancies a) Number of fetus (twin, triplet, quadruplets)

b) Number of fertilized egg (zygosity; mono,di)

c) Number of placenta (chorionicity) d) Number of amniotic cavity (amnionicity)

Complication of multiple pregnancies

To the mother

1) Hyperemesis gravidarum and in fact all physiological response towards pregnancy will be exaggerated.

2) Exacerbation of chronic illness 3) Severe anemia in pregnancy 4) DIC secondary to fetal death. 5) Miscarriage 6) Preterm labour. 7) Polyhydramnios 8) Pre eclampsia. 9) Placenta abruptio 10) Post partum hemorrhage. 11) Higher risk for developing GDM

Others: acute fatty liver, pulmonary embolism

To the fetus

1) IUD of one fetus 2) IUGR 3) Fetal abnormality 4) Pre term delivery 5) Low birth weight 6) Acute respiratory distress syndrome. 7) Congenital abnormality (mental

retardation, Siamese twin, cerebral palsy) Others: twin-to-twin transfusion syndrome,

Twin reversed arterial perfusion (TRAP)/ acardiac twinning

Page 45: Long case examination for phase iii medical students usmkk

Long Case Examination for Phase III Medical Students University Science Malaysia

38

29 Years old Malay lady, G1P0, twin pregnancy at 24/52 was admitted because of frothy color urine and headache. Questions

1) What is your provisional diagnosis and justify your answer?

2) What is twin to twin transfusion syndrome?

3) Factors contributes to preterm delivery 4) How do you manage this patient?

Answer

Provisional diagnosis Pre eclampsia

- Occur at 24w of gestation. - Primigravida - Twin pregnancy - Symptoms of impending pre eclampsia

(frothy urine suggestive of proteinuria and headache.)

Twin to twin transfusion syndrome

- Intrauterine blood transfusion from

donor twin to another recipient twin - Donor twin has smaller size and anemic - Recipient twin will be plethoric - Only occur in monozygotic twin with

monochorionic placenta

Factors contributes to preterm delivery

- Lower and upper genital tract infection - Uterine over distension - Cervical incompetence - Maternal medical complications,

maternal stress - Fetal, placental or uterine abnormalities

Management to this patient 1) Evaluation on severity of the pre

eclampsia - Close monitoring of blood pressure (15

minutes interval until BP stable) - Repeat Dipstick testing within 6H - 24 hour urinary protein - PE Profile (platelet count, uric acid

level, sr Creatinine level, liver enxyme) - Clotting study if platelet < 100 x 106/l 2) Management of hypertension

3) Fetal surveillance - CTG for fetal well being. - Biophysical profile (Ultrasound

monitoring of fetal movement, fetal tone and fetal breathing, ultrasound assessment of liquor volume with or without assessment of fetal heart rate)

4) Anticipating in preterm delivery by

giving IM Dexamethasone, 12 MG, and 12 hours apart.

5) Others - Bed rest - Reduce physical activity - Reduce high cholesterol and salty diet.

Mild PE T. Methyldopa 250mg tds, max 3g/day or T. Labetolol 100 mg tds, max 300mg tds Or, Tab. Nifedipine 10 mg tds stat dose

Severe PE

IV hydrallazine start 5mg, double if no effect until 35mg. change drug if fails or IV Labetolol start 10 mg, double if no effect until max 300mg/day)

** MgSo4 slow infusion 4g 10-15 minutes. Maintenance dose IV ig/hour

Page 46: Long case examination for phase iii medical students usmkk

Long Case Examination for Phase III Medical Students University Science Malaysia

39

Level B Evidence Tocolytic agents should be used judiciously in multiple gestations Women with high-order multiple gestations should be queried about nausea, epigastric pain and other unusual 3rd-trimester symptoms because they are at increased risk to develop HELLP syndrome, in many cases before symptoms of preeclampsia have appeared. The higher incidence of gestational diabetes and hypertension in high-order multiple gestations warrants screening and monitoring for these complication. Level C (expert opinion) The national Institutes of Health recommends that women in preterm labor with no contraindication to steroid use be given one course of steroids regardless of the number of fetuses

Cerclage, hospitalization, bed rest, or home uterine activity monitoring has not been studied in high order multiple gestations, and, therefore should not be ordered prophylactically. There currently is no evidence that their prophylactic use improves outcome in these pregnancies Because the risks of invasive prenatal diagnosis procedures such as amniocentesis and chorionic villus sampling are inversely proportional to the experience of the operator, only experienced clinicians should perform these procedures in high-order multiple gestation. Women should be counseled about the risks of high order multiple gestation before beginning ART Management of discordant growth restriction of death of one fetus in a high-order multiple gestations should be individualized, taking into consideration the welfare of the other fetus (es)

Multiple Pregnancies Summary of recommendation from Clinical Management Guidelines for Obstetrician-Gynecologists Number 56, October 2004

Page 47: Long case examination for phase iii medical students usmkk

Long Case Examination for Phase III Medical Students University Science Malaysia

40

36 years old Malay lady, housewife, G5P4 at 30W+ 5/7 POA with known history of chronic rheumatic heart disease and on single drug therapy presented with sudden onset shortness of breath on the day of admission, cannot lie flat on night and reduce effort tolerance. There was no sign and symptoms suggestive of lung infection. Currently she is not in labour and fetal movement is good. Questions

1) What sign and symptoms that you would like to elicit to suggest that this is a case of heart failure

2) How do you manage this patient Answer

Sign and symptoms of heart failure Cardiac symptoms: exertional dyspnoea, orthopnea, paroxysmal nocturnal dyspnoea, dyspnoea at rest, acute pulmonary edema, chest pain, palpitation. Non cardiac symptoms: anorexia, nausea, weight loss, bloating, fatigue, weakness, oliguria, nocturnal, and cerebral symptoms

Physical examination: Clubbing, prolong capillary refilling, weak, rapid, and thready pulse, tachycardia, diaphoresis, pallor, peripheral cyanosis with pallor and coldness of the extremities, pulmonary rales, edema, hepatomegally, pleural effusion, ascites, cardiomegally, murmurs.

Management to this patient

Acute management

Similar to managing non pregnant patient where the aim is to stabilize the patient

1. Secure the ABC – airway, breathing and circulation.

2. Prop up the patient 45o. 3. Oxygen 100% 3L/min via nasal prong 4. Intravenous access. 5. Take the blood for arterial blood gas

(ABG). 6. 12 lead ECG. 7. Intravenous furosemide, 40 – 80 mg stat

and maintenance. 8. Intravenous digoxin and intravenous

diamorphine 2.5 – 5 mg slowly – depending on medical request.

Further management:

1. Admit the patient to antenatal ward. 2. Carry out all the investigations as

mentioned above. 3. Continue oxygenation. 4. Prop up the patient 45o. 5. Close monitoring of

a) vital signs b) input output chart c) Cardiotocography ( CTG )

6. Continue IV furosemide. May change to tablet form when necessary.

7. Tab slow potassium. 8. Consider IM dexamethasone 12 mg b.d,

6 hours apart to anticipate pre term delivery.

9. Should be managed together with medical team.

Advice on discharge:

1. Semi bed rest at home and avoid vigorous activity

2. Regular follow up at combined clinic. 3. If the patient develops any symptoms of

urinary tract infection, upper respiratory tract infection or chest infection, come early to the hospital.

4. Advise to deliver in the hospital. 5. Admit when the patient at term OR

admit earlier if the patient has symptoms of HF.

Page 48: Long case examination for phase iii medical students usmkk

Long Case Examination for Phase III Medical Students University Science Malaysia

41

29 years old Malay lady, G3P2 at 27/52 W POA with mitral valve prolapses (not in failure) Questions

1) What is Eisenmenger’s syndrome 2) How do you manage this patient

Answer Heart disease complicates approximately 1% of all pregnancy with chronic rheumatic heart disease is the commonest cause in Malaysia apart from congenital heart disease, cardiomyopathies, myocarditis and coronary artery diseases.

Eisenmenger’s syndrome

Pulmonary hypertension secondary to uncorrected congenital heart disease characterized with right-to-left shunting and the associated cyanosis

Management to this patient Antenatal

1. Should be managed in combined clinic. 2. Advice on

a) Avoid doing vigorous activities b) Avoid from getting infection – i.e.

good oral hygiene, treat UTI or URTI.

3. Medication – T.frusemide, hematinic, folic acid and anticoagulant prophylaxis i.e. LMWH

4. Come to the hospitals if develops any symptoms of heart failure, UTI, URTI or chest infection

5. Advise to deliver in the hospital. 6. Admit when the patient at term OR

admit earlier if the patient has symptoms of HF.

Management of intrapartum:

1. Aim for SVD 2. C-sec if any obstetric problem or in

view of cardiologist that patient cannot

withstand stress of normal labour. The same thing goes for Induction of labour.

3. Patient should be placed on high risk category with present of senior obstetrician and consultation from cardiologist.

4. Close monitoring of vital signs, CTG and oxygen saturation. Each patient must be attended by one staff nurse or mid wife

5. Patient must be in prop up position with oxygen is freely available.

6. Pain management – in form of epidural anesthesia (if the patient has no contraindication) or opiates.

7. Prophylactic antibiotic to give adequate protection: Ampicillin or Gentamicin or Amoxicillin

8. Prolonged second stage labour must be assisted

9. Syntocinon is used in third stage of labour instead of ergometrine or syntometrine.

Management of postpartum:

1. Adequate rest for maternal. 2. Encourage breast feeding unless she

cannot cope with it. 3. Continue oral antibiotic for 5 days. 4. Adequate anti coagulant prophylaxis i.e

warfarin to prevent deep vein thrombosis and thromboembolism.

5. Discussion about family planning. Based on

a. Severity of the heart disease b. Completed family

Methods: a. Hormonal contraception - COC will increase risk of TE. b. IUCD should be discouraged due to risk of infection. c. Sterilization

Page 49: Long case examination for phase iii medical students usmkk

Long Case Examination for Phase III Medical Students University Science Malaysia

42

19 Years old Malay lady, G2P0+1(abortion) at 39 W POA with history of chronic rheumatic heart disease with mitral valve replacement, infective endocarditis and completed treatment and history of overwafarinization at 15W POA currently admitted for elective heparin infusion Questions

1) Should warfarin be used in pregnancy 2) How to access functional cardiac status

during pregnancy 3) Why do you think this patient admitted

for heparin infusion? Answer

Should warfarin be used in pregnancy? Warfarin (Coumadin®) is an oral anticoagulant that inhibits synthesis of vitamin K–dependent clotting factors, including factors II, VII, IX, and X, and the anticoagulant proteins C and S [Shirin Abadi et al] Literature suggests a strong association between maternal warfarin use and fetal adverse effect [Shirin Abadi et al] If possible, warfarin therapy should be avoided during pregnancy. If warfarin therapy is essential, it should be avoided at least during the first trimester (because of teratogenicity) and from about 2 to 4 weeks before delivery to reduce risk of hemorrhagic complications [Shirin Abadi et al] Using warfarin between 6 and 12 weeks’ gestation is associated with “fetal warfarin syndrome,” which is most commonly manifested by nasal hypoplasia, stippled epiphyses, limb deformities, and respiratory distress Other complication includes central nervous system abnormalities (mental retardation,

microcephaly, optic atrophy, and blindness, seizures, Dandy-Walker syndrome, and focal cerebellar atrophy),absent or non-functioning kidneys, anal dysplasia, deafness, hemorrhagic complications, premature births, spontaneous abortions, stillbirths, and death How to access functional cardiac status during

pregnancy

Clinical Practice Guideline on "Management of

Heart failure", KKM

Why do you think this patient admitted for heparin infusion?

Pregnancy itself predispose patient to risk of thromboembolism. Furthermore, this patient has been on prosthetic mitral valve which further increases the risk of thromboembolism. Unfractionated heparin or low molecular weight heparin could be substituted when appropriate because these agents do not cross the placenta and are considered the anticoagulant drugs of choice during pregnancy. [Shirin Abadi et al]

Page 50: Long case examination for phase iii medical students usmkk

Long Case Examination for Phase III Medical Students University Science Malaysia

43

38 Years old Malay lady, housewife, G9P6+2(abortion) at 12/52 POA and known case of Mitral Valve Prolapse with mild Mitral regurgitation for more than 10 years Questions

1) Type of murmur in mitral regurgitation and stenosis and Pathophysiology 2) What is heart failure? 3) Contraindication for pregnancy in heart

disease

Types of murmur and Pathophysiology Mitral regurgitation

- Pan systolic murmur - Occur when ventricles leak to a lower

chamber or vessel because there is pressure gradient from the moment ventricle begin to contract.

- Blood then flow and murmur begin at beginning of first heart and continue until the pressure equalize

- Other causes of pan systolic murmur include tricuspid regurgitation, VSD, and Aortopulmonary shunts.

Mitral stenosis

- Mid diastolic murmur. - Begin later in diastolic and may be short

or extend right up to first heart sound. - Due to impairs flow during ventricular

filling either because of stenosis, or obstruction by tumor mass (atrial myxoma)

- Can also due to Austin Flint murmur of aortic regurgitation and Carey Coombs murmur of acute rheumatic fever,.

What is Heart failure?

Heart failure is a syndrome manifesting as the inability of the heart to fill with or eject blood

due to any structural or functional cardiac conditions [Hunt SA et al] According to European Society of Cardiology, HF is a syndrome whereby the patient should have the following features; typically shortness of breath at rest or exertion, and/fatigue sign, signs of fluid retention and objective evidence of an abnormality of the structure or function of the heart at rest. There have been a confusing in defining the type of heart failure especially involving the acute or chronic state. Furthermore, many clinicians use it interchangeably to refer to severity of the disease. Therefore, the ESC guidelines have come across with new definition which distinguishes between new onset HF, transient HF and chronic HF. New onset HF is self-explanatory and refers to first presentation. Transient HF refer to symptomatic HF over a limited time period, although long-term treatment may be indicated, for examples; patient with mild Myocarditis with nearly complete recovery, patient with MI who needs diuretic in CCU but not require it in long term treatment or transient HF caused by ischemia that resolve with revascularization Meanwhile chronic heart failure is a persistent heart failure with stable, worsening or decompensated state.

Contraindication for pregnancy 1) Pulmonary hypertension 2) Eisenmenger’s Syndrome 3) Aortic dilatation > 4cm and this should

be suspected in Marfan syndrome 4) Mother on warfarin treatment. 5) Severe cyanotic heart disease with low

oxygen saturation and high hematocrit

Page 51: Long case examination for phase iii medical students usmkk

Long Case Examination for Phase III Medical Students University Science Malaysia

44

18 years old Malay lady, G1P0 at 32/52 POA was admitted because of severe lethargy, shortness of breath and light-headedness. Questions

1) Define anemia 2) Common cause of anemia in pregnancy 3) Management to this lady 4) What is haematinic 5) 1 pack cell blood can increase how

much hemoglobin level 6) Complication of anemia

Answer

Anemia Hemoglobin concentration <11.0 g/dL [WHO]

a) Mild (Hb 8-10 g/dL) b) Moderate (Hb 5-8 g/dL) c) Severe (Hb less than 5 g/dL

Common cause of anemia in pregnancy 1) Microcytic anemia (Iron deficiency

anemia. Needs to exclude Thalassemia as both will give low MCV of <85 fL)

2) Macrocytic anemia (folate deficiency) 3) Trauma 4) Hemolytic anemia

a) Sickle cell syndrome b) Sickle cell disease c) Sickle cell traits d) Sickle cell hemoglobin C disease.

Management for this lady

1) Admit to wards for observation 2) Monitoring of vital sign. 3) Full blood count (pay attention on

hemoglobin level for grading and MCV for type of anemia), serum ferritin level, total iron binding capacity.

4) Full blood picture if iron deficiency anemia is unlikely

5) Establish the causes of anemia

6) Excludes the differential diagnosis of lethargy, shortness of breath and light-headedness a) Cardiovascular problem b) Respiratory tract infection c) Multiple pregnancy d) Diabetes mellitus

7) For iron deficiency anemia a) Iron supplement like ferrous

sulphate and ferrous fumarate (increase approximately 1 Hb in 1 week) # may consider double hematinic (doubling the dose)

b) Parenteral iron c) Packed cell transfusion.

8) Maternal transfusion should be considered for fetal indications in cases of severe anemia.

9) Course of treatment should be based on clinical judgment and individualized; period of gestation, severity of anemia, type of anemia.

Haematinic

a) Iron b) Folate c) Vitamin C (increase absorption of iron)

One packed cell

- Is 450 ml of blood. Ideally it will increase 1 Hb level

Complication of anemia

1) To mother a) Aggravate heart failure b) Risk of post partum hemorrhage c) Increase risk of infection

2) To fetus

a) Fetal hypoxia b) IUGR c) Spontaneous abortion.

Page 52: Long case examination for phase iii medical students usmkk

Long Case Examination for Phase III Medical Students University Science Malaysia

45

Anemia: Hgb (g/dL) and Hct (percentage) levels below 11 g/dL and 33%, respectively, in the first trimester; 10.5 g/dL and 32%, respectively, in the second trimester; and 11 g/dL and 33%, respectively, in the third trimester Summary of Recommendations and Conclusions The following conclusion is based on good and Consistent scientific evidence (Level A):

- Iron supplementation decreases the prevalence of maternal anemia at delivery.

The following recommendation and conclusions are based on limited or inconsistent scientific data (Level B):

- Iron deficiency anemia during pregnancy has been associated with an increased risk of low birth weight, preterm delivery, and perinatal mortality. Severe anemia with maternal Hgb levels less than 6 g/dL has been associated with abnormal fetal oxygenation resulting in non reassuring fetal heart rate patterns, reduced amniotic fluid volume, fetal cerebral vasodilatation, and fetal death. Thus, maternal transfusion should be considered for fetal indications.

The following recommendations are based primarily on consensus and expert opinion (Level C):

- All pregnant women should be screened for anemia, and those with iron deficiency anemia should be treated with supplemental iron, in addition to prenatal vitamins.

- Patients with anemia other than iron

deficiency anemia should be further evaluated.

- Failure to respond to iron therapy

should prompt further investigation and may suggest an incorrect diagnosis, coexisting disease, malabsorption (sometimes caused by the use of enteric-coated tablets or concomitant use of antacids), noncompliance, or blood loss.

Clinical Management Guidelines for Obstetrician–Gynecologists Number 95, July 2008

Page 53: Long case examination for phase iii medical students usmkk

Long Case Examination for Phase III Medical Students University Science Malaysia

46

28 Years old Malay lady, housewife, G2P1 at 38W + 5/7 POA was admitted in view of

1. One Previous scar a year ago due to fetal distress, uncomplicated CS

2. Uterine fibroid diagnosed during first pregnancy

Questions

1) Why fibroid increase in size during

pregnancy 2) What fibroid changes can occur during

pregnancy 3) Expected findings during PE 4) How do you manage the pregnancy

patient with fibroids? Answer Why fibroid increase in size during pregnancy

Fibroid is an estrogen-dependant for its growth. During pregnancy, the level of estrogen rise steadily until term.

Fibroid changes during pregnancy

Red degeneration can occurs between 12th and 22nd week of pregnancy where the blood supply to the fibroid is cut off. As a result, the fibroid turn red and die. Red degeneration can cause intense abdominal pains and uterine contraction which could lead to early labour or miscarriage

Expected findings during PE

1) More than 2 fetal pole palpated 2) Some fibroids may cause unstable lie or

breech presentation.

Management of pregnant lady with fibroids

1) Management of fibroids during pregnancy is just a monitoring of its growth. Drugs rarely being prescribed.

2) Fibroids can cause miscarriage. Therefore, advise patient not to undergone vigorous activity.

3) Corticosteroid injection if pre term labour is anticipated.

4) Monitoring of the fetus for fetal well being, lie and presentation.

5) Observation of fetal lie and presentation is crucial before allowing patient to go for SVD.

6) Furthermore, obstructed labour should be excluded.

7) If patient is scheduled for caesarean section, do not remove the fibroid during the surgery as it will cause heavy bleedings.

8) Fibroids will shrink after pregnancy. Management post pregnancy includes a) Ablation of the fibroids b) Surgery to remove the fibroids c) Hysterectomy d) Medications to shrink the fibroids

Page 54: Long case examination for phase iii medical students usmkk

Long Case Examination for Phase III Medical Students University Science Malaysia

47

16 years old Malay lady, single parent with G1P0 at 37 w + 6/7 POA with history of

1) Bronchial asthma on MDI salbutamol and MDI inflammide

2) Anemia on double hematinic. History of blood transfusion at 26/52

3) Ante partum hemorrhage secondary to PP type I at 34/52. Completed dexamethasone.

4) Currently admitted because of contraction pain.

5) Pre marital sex

Questions

1) What is the effect of Salbutamol (ventolin) on uterine contraction

2) Complication of teenage pregnancy 3) Principles of Dexamethasone and

dosage.

Answer

Salbutamol action on uterus

Salbutamol stimulates beta 2 receptor in the uterus and causing muscles in the wall of uterus to relax.

Complication of teenage pregnancy

Teenage pregnancy is defined as a teenage girl, usually within the ages of 13-19, becoming pregnant. [UNICEF]

a) Highest global incidence for premature birth and low birth weight

b) High risk for anemia in pregnancy c) Difficulties in labour due to

underdeveloped pelvis d) High risk for obstructed labour, causing

obstetric fistula if c-sec is not readily accessible

e) Leading causes of death for girls aged 15 to 19 in developing countries

f) Predispose to un safe abortion g) High risk for HIV infection due to

unprotected sex

Dexamethasone

Pre term labor associated with complication of respiratory distress syndrome, intraventricular hemorrhage and necrotizing enterocolitis. Multiple randomized controlled trials has demonstrated that the admission of corticosteroid to the mother resulting significant reduction in these complication.

Both betamethasone and dexamethasone can cross the placenta. In USM, dexamethasone is used instead of betamethasone. Betamethasone may be a better choice because it can reduce the risk of cystic periventricular leukomalacia.

It can be given in doses of 6 mg every 12 hours for four doses, and 12 mg every 12 hours for 2 doses during period of gestation of 24 W to 34 through IM route.

In term of lung maturity, no benefit has been demonstrated in infants beyond 34 weeks gestation. However, it is still being given in many centers because of no harm to the baby and mother. Furthermore, it still can improve the complication of intraventricular hemorrhage and necrotizing enterocolitis.

Beware when used in mother with poorly controlled diabetes in pregnancy, chorioamnionitis and immunosuppressed mothers

Corticosteroid shows maximum effect after 24 hour and lasted 7 days. Repeated use of corticosteroid should be avoided as it may impose complication to mother and fetus.

Page 55: Long case examination for phase iii medical students usmkk

Long Case Examination for Phase III Medical Students University Science Malaysia

48

33 Years old Malay lady, housewife, G7P6 at 38 W POA was admitted for further management in view of

1) Grand multi para 2) Maternal obesity but MOGTT test is

normal 3) Placenta previa type III anterior but no

history of APH. 4) Biggest baby is 3.9 kg. 5) Clinically, suspected macrosomic baby.

On ultrasound, estimated fetal weight is 4.0 to 4.2 kg.

6) Not in labour yet.

Question

1) What is the mode of delivery and justify your answer

2) How do you manage this patient

Answer

Mode of delivery

After thorough view on this patient presentation, caesarian section is the most appropriate mode of delivery because of

1) Placenta previa type III in which descending of fetal head into pelvic brim is impossible due to obstruction to cervical os.

2) Maternal obesity and macrosomic baby will predispose to complication especially shoulder dystocia.

3) Induction of labour also impossible because of grand multi para with macrosomic baby.

Management to this patient

Ante natal management

1) Daily CTG for fetal surveillance. Ultrasound should also be done.

2) Counseling on mode of delivery and reason why patient can not be allowed for SVD.

3) Plan for Caesarean section. Can be performed at 38 week to 39 week of gestation.

4) Counseling for bi tubal ligation because patient is in high risk to develop uterine rupture, hemorrhage and uterine atony after this c-sec delivery due to a) Grand multi para b) 1 c-sec scar.

Intrapartum management

1) Preparation for c-sec (refer c-sec preparation)

2) Blood GSH because anticipating in blood loss because we will cut through the placenta.

3) Presence of senior obstetrician in case of complication to mother during operation and pediatrician for management of baby.

Post partum

1) Baby should be check for capillary blood sugar and early feeding is encouraged

2) Baby should be managed by pediatrician and kept in NICU for observation.

3) Mother should be offered with other type of contraception if she refused bi tubal ligation.

4) Daily inspection on c-sec scars to look for any infection or ruptured scar.

5) Referral to the mother to dietitian and internal medicine team for further management on obesity.

6) Mother should be offered MOGTT screening on the next pregnancy.

Page 56: Long case examination for phase iii medical students usmkk

Long Case Examination for Phase III Medical Students University Science Malaysia

49

40 Years old Malay lady, housewife, G15P12+2 (abortion) at 36W + 3/7 from low socioeconomic status presented with this problem list

1) Elderly pregnancy 2) Great grand Multipara (para >10) 3) Unstable lie

Questions

1) Complication associated with great grand Multipara

2) Principles of management in this patient 3) What is the contraindication for ECV?

Answer

Notes: The risk of cord prolapses leading to fetal hypoxia and fetal death is very high once the membrane ruptures. Therefore, it is highly recommended that patient with unstable lie should be admitted to antenatal wards at 37 weeks onward for observation [The Practical Labour suite Management]

Complication associated with great grand Multipara

The incidence of malpresentation at the time of delivery, maternal obesity, anemia, preterm delivery, and meconium-stained amniotic fluid increased with higher parity, whereas the rate of excessive weight gain and cesarean delivery decreased. Compared with grand multiparas, great-grand multiparas had significantly elevated risks for abnormal amounts of amniotic fluid, abruptio placentae, neonatal tachypnea, and malformations but lower rates of placenta previa (P < .05). The incidence of postpartum hemorrhage, preeclampsia, placenta previa, macrosomia, postdate pregnancy, and low Apgar scores was significantly higher in grand multiparas than in multiparas, whereas the proportion of induction, forceps delivery, and total labor complications was significantly lower

than in the multiparous group (P < .05). Similar frequency of maternal diabetes, infection, uterine wall scar rupture, variations in fetal heart rate, fetal death, and neonatal mortality was found in the 3 groups. [Agota Babinszki et al]

Principles of management in this patient

1) Daily observation for fetal lie 2) Exclude factors contributing to unstable

lie such as fetal abnormality, placenta previa, uterine abnormality

3) Pelvic abnormality 4) Polyhydramnios 5) Discuss on mode of delivery which is

expectant vs. active management (C-sec, ECV or stabilizing induction)

6) Advise on contraception. Tubal ligation should be offered in view of a) Advanced maternal age b) Great grand multi parity c) Two history of abortion. d) Low socio economic status.

� As a Muslim, permanent method of contraception should only be performed if specialist agrees that next pregnancy will be harmful to the patient’s life.

Contra indication for ECV

Absolute contraindication 1) Multiple pregnancy 2) APH 3) PP 4) PROM or PPROM 5) Significant fetal abnormality 6) Any indication for caesarean section.

Relative 1) Previous c-sec 2) IUGR 3) Severe proteinuric hypertension 4) Rhesus iso-immunization 5) Evidence of macrosomia 6) Any suspected fetal compromise.

Page 57: Long case examination for phase iii medical students usmkk

Long Case Examination for Phase III Medical Students University Science Malaysia

50

26 Years old Malay lady, G1P0 at 38 W POG currently presented with this problem

1. Extended breech 2. Persistent proteinuria 3. Hypothyroidism on Tab Thyroxine 50

microgram OD and undergone total thyroidectomy in 2002

Questions

1) Hormones affecting level of thyroid hormone during pregnancy.

2) What is the effect of hypothyroidism in pregnancy

3) Management to this lady Answer Hormones interacting with thyroid hormones

1) High level of human chorionic gonadotropin will decrease the level of TSH during first trimester

2) Estrogen will increases the amount of thyroid hormone binding proteins in the serum hence increases the total thyroid hormone levels in the blood. However, free hormone remains normal

Effect of hypothyroidism on pregnancy

1) Mother

a) No symptoms in mild hypothyroidism

b) Maternal anemia c) Maternal myopathy d) Congestive heart failure e) Pre eclampsia f) Placental abnormalities g) Low birth weight infants h) post partum hemorrhage

2) Baby

a) Severe cognitive, neurological and developmental activity

Management to this lady

1) The management in view of breech is

similar to normal pregnancy 2) Excludes the causes of proteinuria. Take

notes that primid with proteinuria and hypothyroidism should increase suspiciousness to pre eclampsia even though it is already near term.

3) For the management of hypothyroidism a) WHO recommends intake of 200

micrograms/day of iodine during pregnancy to maintain adequate thyroid hormone production

b) Dosage of maintenance thyroxine could be increased during pregnancy up to 50-200 microgram daily dose. (Doubling the dose by 25% to 50%)

c) Thyroid function tests every 6-8 weeks during pregnancy to ensure normal thyroid function throughout pregnancy.

4) Screening for congenital hypothyroidism to the baby using the umbilical cord blood during delivery.

Page 58: Long case examination for phase iii medical students usmkk

In Pursuit to Excel MCQ Exam for Professional III Examination (MCQ)

1

Asherman’s syndrome (intrauterine adhesion) is associated with:

A) Amenorrhea B) Placenta previa C) Subfertility D) Salphingitis E) Menorrhagia

History: Named after Dr Asherman, an Israeli gynaecologist, who first described the condition in the mid 20th century when he noted that some women who had surgical treatments at the time of pregnancy stopped having periods after this treatment

Def: A reduction or absence in menstruation that may be due to scar tissue formation inside the uterus and can occur as a result of pregnancy and delivery, infection or gynaecological surgical procedure

**differ from endometrial ablation: induce scar tissue in the uterus to prevent heavy periods

Classification according to 17th Congress of the Federation of Frenchspeaking Societies of Gynaecology and Obstetrics [1957]

(1) Traumatic synechiae connected with surgical or obstetrical evacuation of the uterus. (2) Spontaneous synechiae of tuberculous origin. (3) Synechiae occurring after myomectomy. (4) Synechiae secondary to the attack of chemical or physical agents and, likewise, those resulting fromatrophic changes. Pathophysiology: Severely damaged decidua basalis are replaced with granulation tissue and opposing uterine wall adhere to form scar tissue. It is later infiltrated by myometrial cells and covered by endometrium.

Fibroids, due to their mere presence, cause the entire uterus to enlarge, thereby stretching the blood vessels that supply the various parts of the uterus. When the fibroids are removed, the remaining uterus collapses down to a smaller volume and some of the blood vessels that supply the endometrium may become blocked. Because of the lack of oxygen and nutrients, that area of tissue may die and a scar may form leading to Asherman’s syndrome. Other cause: radiation cause ischemia to myometrium tissue Signs and symptoms 1) Oligomenorrhea, amenorrhea 2) Pain (increase work by uterine muscle to get rid blood through scar tissue) 3) Infertility 4) Haematometra (large bruise inside uterus that diagnosed by pelvic US) Complication

1) Placenta previa, accrete 2) Infertility 3) IUGR 4) Ectopic pregnancy

Investigation: 1) Hysteroscopy 2) Hysterosalpingogram (HSG)

Management

1) Admission to wards 2) Investigation to confirm the diagnosis 3) Surgical excision of scar tissue by

hysteroscope under General anesthesia. Answer: T, T, T, F, F

Page 59: Long case examination for phase iii medical students usmkk

In Pursuit to Excel MCQ Exam for Professional III Examination (MCQ)

2

Concerning heart disease in pregnancy A. Mitral stenosis carries a better

prognosis than atrial defect B. Pregnancy should be induced at 38

weeks in cases with grade IV dyspnoea

C. Rheumatic heart disease is more common than congenital heart disease

D. Ergometrine should be avoided in most cases

E. Mitral volvotomy is contraindicated. Introduction: About 0.5 – 4% of pregnant women have cardiac disease. Common causes of HF in pregnancy are hypertension, eclampsia, undetected valvular heart disease especially mitral stenosis, congenital heart disease, and occasionally peripartum cardiomyopathy. Peripartum cardiomyopathy occurs in 1:3,000 life births in Malaysia [Management of HF Malaysia] NYHA classification did not show any marked differences in outcome. Eismenger’s syndrome and pulmonary hypertension carries 40-50% mortality rate (can be caused by mitral stenosis). TOF 5% if no pulmonary HPT. Normal haemodynamic changes that occur in pregnancy are: 1) Cardiac output increases by 30–50% during normal pregnancy. 2) Cardiac output increases to 80% above baseline during labour and delivery. **Haemodynamic changes return to baseline 2 – 4 weeks after vaginal delivery and up to 6 weeks after caesarian delivery. Management 1) Manage by multidisciplinary team consist of physician, obstetrician and pediatrician

a. Non pharmacological (mild) - limiting strenuous exercise - adequate rest - maintaining a low salt diet - treating anemia and infections early - frequent antenatal examinations Pharmacological - Sublingual GTN - Digoxin - Diuretic(used with care as may impair uterine blood flow. No teratogenic effect) - Beta blocker (used with care;intrauterine growth retardation, apnea at birth,fatal bradycardia, hypoglycaemia and hyperbilirubinemia) - ACEI and ARB are contraindicated in pregnancy. -warfarin is teratogenic in early trimester. Heparin can be used LMW subcutaneous. Labour is spontaneous except in fetal compromise (consider pre mature delivery) Epidural anaesthesia during labour is recommended. Ergometrine causes 1. Vasoconstriction, HPT and heart failure. So must be avoided. Used syntocinon only. Antibiotic prophylaxis in structural heart abnormality during labour (IV ampiillin 1.0g 6h X 3doses, IV gentamycin 80 mg 8 hourly X 3) - Surgical volvotomy ideally be performed before pregnancy although it is safe to do it during pregnancy - Marfan syndrome is an autosomal dominant connective tissue abnormality that may lead to mitral valve prolapsed and aortic regurgitation, aortic root dilatation and aortic rupture (50% in pregnancy) Answer: F, F, T, T, F

Page 60: Long case examination for phase iii medical students usmkk

In Pursuit to Excel MCQ Exam for Professional III Examination (MCQ)

3

Pyelonephritis in pregnancy A. Occur in 0.1% of pregnant woman B. If unilateral, is most often right sided C. Caused predominantly by staph.

Aureus D. Common in diabetic patient

Def: Pyelonephritis is the most common urinary tract complication in pregnant women, occurring in approximately 2% of all pregnancies. Symptoms of pyelonephritis include the following: 1. Fever (Often, the temperature is very high.) 2. Chills 3. Nausea and vomiting 4. Costovertebral angle (CVA) or flank pain Flank tenderness is right-sided in more than half of patients, bilateral in one fourth of patients, and left-sided in one fourth of patients. Pain may also be found suprapubically with palpation.

Other symptoms may include nausea, vomiting, frequency, urgency, and dysuria. Women with additional risk factors (immunosuppression, diabetes, sickle cell anemia, neurogenic bladder, recurrent or persistent UTIs prior to pregnancy) are at an increased risk of a complicated UTI Common organism

1. Escherichia coli (most common, in as many as 70% of cases)

2. Group B Streptococcus (10%) 3. Klebsiella or Enterobacter species (3%) 4. Proteus species (2%)

Physiological changes include urinary retention caused by the weight of the enlarging uterus and urinary stasis due to ureteral smooth muscle relaxation

Investigation 1) Full blood count 2) Urinalysis:Positive results for nitrites, leukocyte esterase, WBCs, RBCs, and protein suggest UTI. 3) Urine culture: A colony count of 100,000 colony-forming units (CFUs) per milliliter has historically been used to define a positive culture result 4) Renal ultrasonography 5) Evaluation of fetal status 6) Renal function test Treatment -Ampicillin 2 g IV q6h for treatment of pyelonephritis; use in conjunction with an aminoglycoside for treatment of pyelonephritis -Paracetamol -Amoxicillin 7-Day regimen: 250 mg PO q8h or 3-Day regimen: 500 mg PO qid -Amoxicillin/clavulanate potassium (Augmentin), 500 mg PO tid for 7-10 d -Ceftriaxone (Rocephin) Complication - Bacteremia - Respiratory insufficiency due to bacterial endotoxin damage to the alveoli, causing pulmonary edema; therefore, fluid overload - Renal dysfunction - PPROM - Pre term birth Answer; F, T, F, T

Page 61: Long case examination for phase iii medical students usmkk

In Pursuit to Excel MCQ Exam for Professional III Examination (MCQ)

4

STD include A. Trichomonas vaginitis B. Condyloma accuminata C. Chlamydial infection D. Type 1 herpes hominis E. Toxoplasmosis

Bacterial STDs include syphilis, gonorrhea, chancroid, lymphogranuloma venereum, granuloma inguinale, and chlamydial, mycoplasmal, and Ureaplasma infections. Viral STDs include genital and anorectal warts (Condylomata acuminata :HPV types 6 and 11), genital herpes, molluscum contagiosum, and HIV infection Parasitic infections that can be sexually transmitted include trichomoniasis (caused by protozoa), scabies (caused by mites), and pediculosis pubis (caused by lice). Type 1 herpes hominis causes classic “cold sores” or “fever blisters”, commonly known as herpes simplex, herpes genitals, herpes labialis Toxoplasmosis is caused by infection with Toxoplasma gondii, an obligate intracellular parasite. The infection produces a wide range of clinical syndromes in humans, land and sea mammals, and various bird species.Individuals at risk for toxoplasmosis include fetuses, newborns, and immunologically impaired patients. Congenital toxoplasmosis is usually a subclinical infection. Among immunodeficient individuals, toxoplasmosis most often occurs in those with defects of T-cell–mediated immunity, such as those with hematologic malignancies, bone marrow and solid organ transplants, or AIDS.T gondii oocysts are ingested in material contaminated by feces from infected cats. Oocysts may also be transported to food by flies and cockroaches

Answer: T, T, T, F, F

Page 62: Long case examination for phase iii medical students usmkk

In Pursuit to Excel MCQ Exam for Professional III Examination (MCQ)

5

Uterine rupture may be associated with A. Previous c-section B. Myomectomy C. Oxytoxin infusion D. Prostaglandin administration E. Breech extraction

Uterine rupture in pregnancy is a rare and often catastrophic complication with a high incidence of fetal and maternal morbidity. Several factors are known to increase the risk of uterine rupture, but, even in high-risk subgroups, the overall incidence of uterine rupture is low. From 1976-2005, 19 peer-reviewed publications that described the incidence of uterine rupture reported 1654 cases of uterine rupture among 2,504,456 pregnant women, yielding an overall rupture rate of 1 in 1514 pregnancies (0.07%). Uterine rupture is defined as a full-thickness separation of the uterine wall and the overlying serosa. Differs from uterine scar dehiscence: Separation of a preexisting scar that does not disrupt the overlying visceral peritoneum (uterine serosa) and that does not significantly bleed from its edges. In addition, the fetus, placenta, and umbilical cord must be contained within the uterine cavity, without a need for cesarean delivery because of fetal distress. Uterine rupture results in:

• bleeding; • rupture of the amniotic sac (bag of

waters); • partial or full delivery of the fetus into

the abdominal cavity; and • loss of oxygen delivery to the fetus.

Classic symptoms of rupture include:

• pain above and beyond normal labor pain;

• discontinuation of uterine contractions; • signs of fetal heart rate abnormalities; • hemorrhage; and • Hypovolumic shock

Consequences of Uterine rupture in fetal 1) Fetal hypoxia or anoxia 2) Fetal acidosis 3) Admission to a NICU 4) Fetal or neonatal death

Consequences of Uterine rupture in mother

1) Maternal bladder injury 2) Severe maternal blood loss or anemia 3) Hypovolemic shock 4) Need for hysterectomy 5) Maternal death

Risk Factor

1) Previous cesarean delivery 2) Previous myomectomy 3) Congenital uterine anomaly 4) Pregnancy considerations

Grand multiparity, Maternal age, Placentation (accreta, percreta, increta, previa, abruption) Cornual (or angular) pregnancy Overdistension (multiple gestation, polyhydramnios) Dystocia (fetal macrosomia, contracted pelvis) Trophoblastic invasion of the myometrium

5) Labor status Induced labor + With oxytocin + With prostaglandins Augmentation of labor with oxytocin Duration of labor, Obstructed labor

6) Obstetric management considerations Instrumentation (forceps use) Intrauterine manipulation (external cephalic version, internal podalic version, breech extraction, shoulder dystocia, manual extraction of placenta) Fundal pressure

7) Uterine trauma Direct uterine trauma and Violence Answer: All True

Page 63: Long case examination for phase iii medical students usmkk

In Pursuit to Excel MCQ Exam for Professional III Examination (MCQ)

6

Labour may be obstructed by A) Ovarian tumor B) Cystocoele C) Ectopic kidney D) Distended bladder E) Vaginal septum

Obstructed labour means that, in spite of strong contractions of the uterus, the fetus cannot descend through the pelvis because there is an insurmountable barrier preventing its descent. Obstruction usually occurs at the pelvic brim, but occasionally it may occur in the cavity or at the outlet of the pelvis. [WHO] Causes of obstructed labour:

1) cephalopelvic disproportion (small pelvis or large fetus)

2) abnormal presentations, e.g. - brow - shoulder - face with chin posterior - aftercoming head in breech presentation 3) Fetal abnormalities, e.g. - hydrocephalus* - locked twins* 4) abnormalities of the reproductive tract, e.g.

- pelvic tumour* - stenosis of cervix or vagina** - tight perineum.**

* Rarer causes. ** This may be associated with scarring caused by female genital mutilation, or previous “gishiri” cut. Answer: All true

Page 64: Long case examination for phase iii medical students usmkk

In Pursuit to Excel MCQ Exam for Professional III Examination (MCQ)

7

During pregnancy and puerperium, fibroid A) increase in size B) Undergo red degeneration C) Become infected D) May undergo sarcomatous changes E) Cause post partum hemorrhage

Uterine leiomyomas, commonly known as fibroids, are well-circumscribed, non-cancerous tumors arising from the myometrium (smooth muscle layer) of the uterus. In addition to smooth muscle, leiomyomas are also composed of extracellular matrix (i.e., collagen, proteoglycan, fibronectin). Other names for these tumors include fibromyomas, fibromas, myofibromas, and myomas. Leiomyomas are usually detected in women in their 30's and 40's and will shrink after menopause in the absence of post-menopausal estrogen replacement therapy. (Dependent on estrogen for growth) Two to five times more prevalent in black women than white women Leiomyomas are classified by their location in the uterus. Subserosal leiomyomas are located just under the uterine serosa and may be pedunculated (attached to the corpus by a narrow stalk) or sessile (broad-based). Intramural leiomyomas are found predominantly within the thick myometrium but may distort the uterine cavity or cause an irregular external uterine contour. Submucous leiomyomas are located just under the uterine mucosa (endometrium) and, like subserosal leiomyomas, may be either pedunculated or sessile. Tumors in subserosal and intramural locations comprise the majority (95%) of all leiomyomas; submucous leiomyomas make up the remaining 5%. Others; Cervical, Intraligamentary (within broad ligament, cause uteric compression) and Parasitic(attached outside the uterus, i.e. the bladder) Transformation of uterine leiomyomas (benign) to uterine leiomyosarcomas (malignant smooth muscle tumors of the uterus) is extremely rare, and, in fact, many researchers and clinicians believe this type of transformation never occurs.

However, without pathologic examination of the uterus, this determination is not possible. Uterine leiomyosarcomas are found in approximately 0.1% of women with leiomyomas and are reported to be more frequently associated with large or rapidly growing fibroids. The two most common symptoms of fibroids (also called leiomyomas) are abnormal uterine bleeding and pelvic pressure. Leiomyomas are also associated with a range of reproductive dysfunction including recurrent miscarriage, infertility, premature labor, fetal malpresentations, and complications of labor. Diagnosis

1. bimanual pelvic examination 2. ultrasonography, MRI (magnetic

resonance imagery), and CT 3. Hysterosalpingography,

sonohysterography, and hysteroscopy Red degeneration: obsolete term for necrosis, with staining by hemoglobin, which may occur in uterine myomas, especially during pregnancy; marked by softening and a red color resembling partly cooked meat. [stedman] Medical treatment

1) NSAIDS for dysmenorrhea 2) antifibrotic drug , pirfenidone 3) GnRH agonist (Specifically, uterine

volume has been shown to decrease approximately 50% after three months of GnRH agonist therapy.) - Use to reduce fibroid size few

months before surgery or - When menopause is within few

months Surgical treatment Myomectomy (pt wish to reproduce) Hysterectomy Answer: T, T, F, F, T

Page 65: Long case examination for phase iii medical students usmkk

In Pursuit to Excel MCQ Exam for Professional III Examination (MCQ)

8

Placenta previa is associated with A) Painless vagina bleeding B) Abnormal fetal heart rate C) Twin pregnancy D) Android pelvis E) Diabetes Melitus

Placenta previa involves implantation of the placenta over the internal cervical os. Variants include complete implantation over the os (complete placenta previa), a placental edge partially covering the os (partial placenta previa) or the placenta approaching the border of the os (marginal placenta previa). A low-lying placenta implants in the caudad one half to one third of the uterus or within 2-3 cm from the os. A leading cause of third trimester hemorrhage, placenta previa presents classically as painless bleeding. Bleeding is thought to occur in association with the development of the lower uterine segment in the third trimester. Placental attachment is disrupted as this area gradually thins in preparation for the onset of labor. When this occurs, bleeding occurs at the implantation site as the uterus is unable to contract adequately and stop the flow of blood from the open vessels. Thrombin release from the bleeding sites promotes uterine contractions and a vicious cycle of bleeding-contractions-placental separation-bleeding. Placental migration occurs during the second and third trimesters, owing to the development of the lower uterine segment, but it is less likely if the placenta is posterior or if there has been a previous caesarean section. Causes * Hemorrhaging, if associated with labor, would be secondary to cervical dilatation and disruption of the placental implantation from the cervix and lower uterine segment. The lower uterine segment is inefficient in contracting and thus cannot constrict vessels as in the uterine corpus, resulting in continued bleeding. * Advancing age (>35) * Multiparity * Infertility treatment

* Multiple gestation (larger surface area of the placenta) * Erythroblastosis * Prior uterine surgery * Recurrent abortions * Nonwhite ethnicity * Low socioeconomic status * Short interpregnancy interval * Smoking * Cocaine use * Other causes include digital exam, abruption (pre-eclampsia, chronic hypertension, cocaine use, etc) and other causes of trauma (eg, postcoital trauma).

Grade.

I Placenta encroaches on the lower uterine segment but does not reach the cervical os.

Lateral Minor

II Placenta reaches the margin of the cervical os but does not cover it.

marginal major if posterior located

III Placenta partially cover the os.

IV Placenta is symmetrically implanted in the lower uterine segment

Complete Major

Answer: T, F, T, F, F

Page 66: Long case examination for phase iii medical students usmkk

In Pursuit to Excel MCQ Exam for Professional III Examination (MCQ)

9

Complication of abruptio placenta A) Eclampsia B) Acute renal failure C) DIVC D) Fetal death E) Post partum hemorrhage

Abruptio placentae is defined as the premature separation of the placenta from the uterus. Patients with abruptio placentae typically present with bleeding, uterine contractions, and fetal distress. A significant cause of third-trimester bleeding associated with both fetal and maternal morbidity and mortality, abruptio placentae must be considered whenever bleeding is encountered in the second half of pregnancy. Hemorrhage into the decidua basalis occurs as the placenta separates from the uterus. Vaginal bleeding usually follows, although the presence of a concealed hemorrhage in which the blood pools behind the placenta is possible. If the bleeding continues, fetal and maternal distress may develop. Fetal and maternal death may occur if appropriate interventions are not undertaken. The primary cause of placental abruption is usually unknown, but multiple risk factors have been identified. Complication Cesarean delivery: Cesarean delivery is often necessary if the patient is far from her delivery date or if significant fetal compromise develops. If significant placental separation is present, the fetal heart rate tracing typically shows evidence of fetal decelerations and even persistent fetal bradycardia. A cesarean delivery may be complicated by infection, additional hemorrhage, the need for transfusion of blood products, injury of the maternal bowel or bladder, and/or hysterectomy for uncontrollable hemorrhage. In rare cases, death occurs. Hemorrhage/coagulopathy: Disseminated intravascular coagulation (DIC) may occur as a sequela of placental abruption. Patients with a placental abruption are at higher risk of developing a coagulopathic state than those with placental previa. The coagulopathy must be

corrected to ensure adequate hemostasis in the case of a cesarean Prematurity: Delivery is required in cases of severe abruption or when significant fetal or maternal distress occurs, even in the setting of profound prematurity. In some cases, immediate delivery is the only option, even before the administration of corticosteroid therapy in these premature infants. All other problems and complications associated with a premature infant are also possible. Signs and symptoms

1) Vaginal bleeding - Vaginal bleeding is present in 80%

of patients diagnosed with placental abruptions.

- Bleeding may be significant enough to jeopardize both fetal and maternal health in a relatively short period.

- Remember that 20% of abruptions are associated with a concealed hemorrhage and the absence of vaginal bleeding does not exclude a diagnosis of abruptio placentae.

2) Contractions/uterine tenderness - Contractions and uterine hypertonus

are part of the classic triad observed with placental abruption.

- Uterine activity is a sensitive marker of abruption and, in the absence of vaginal bleeding, should suggest the possibility of an abruption, especially after some form of trauma or in a patient with multiple risk factors.

3) Decreased fetal movement - This may be the presenting

complaint. - Decreased fetal movement may be

due to fetal jeopardy or death. Answer: F(risk factor), T, T, T, T(uterine atony)

Page 67: Long case examination for phase iii medical students usmkk

In Pursuit to Excel MCQ Exam for Professional III Examination (MCQ)

10

A patient with severe placental abruption will need

A) CVP line B) Artificial Rupture Of Membrane C) Sedation with morphine D) Beta adrenergic drug E) Arterial Blood Gas analysis

Sher Severity Grading system 1. Grade 1: (Herald bleed) 1. Less than 100cc of uterine bleeding 2. Uterus non-tender 3. No Fetal Distress 2. Grade 2 1. Uterus tender 2. Fetal Distress 3. Concealed hemorrhage 4. Progresses to Grade 3 without delivery 3. Grade 3 1. Fetal death 2. Maternal shock 3. Extensive concealed hemorrhage 4. Coagulopathy 1. Absent: 3A (66% of patients) 2. Present: 3B (33% of patients) Management: Stable patient (Grade I) A. General 1. Obstetrics Consultation 2. RhoGAM if Maternal blood Rh -ve B. Criteria 1. Reassuring Fetal Heart Tracing 2. No coagulopathy 3. Normotensive without Preeclampsia 4. Nontender uterus 5. Negative ultrasound with normal AFI C. Preterm gestation 1. Consider Tocolysis with Magnesium Sulfate. Contraindicated in all but mild abruption <34 weeks**Controversial and risky 2. Steroids to promote lung maturity 3. Consider Amniocentesis for lung maturity studies 4. External Fetal Monitoring 5. Observe during short term hospitalization D. Term gestation or mature lung studies 1. Active management labor towards rapid fetal delivery

2. Early Rupture of Membranes (AROM) 3. Internal Fetal Monitoring (fetal scalp electrode) 4. Tocometry 5. Intrauterine Pressure Catheter 6. Cautious use of Pitocin E. Risks 1. Preterm birth 2. Intrauterine Growth Retardation Management: Emergent Rapid management is critical as fetal death occur in up to 30% within 2h. Do not wait for US as it is clinically diagnosed 1. Brisk bleeding 2. Unstable vital signs 3. Fetal Distress 4. Grade II or III placental abruption Immediate interventions 1. Oxygen 2. Trendelenburg position 3. Obtain immediate Intravenous Access 1. Two large bore IV (16-18 gauge) 2. Initiate Isotonic crystalloid bolus 1. Normal saline or Ringers 4. Call for immediate Obstetric and neonatal support 5. Delivery within 20 minutes if Fetal Distress** Cesarean Section unless imminent Vaginal Delivery 6. RhoGAM if Maternal blood Rh -ve Monitoring 1. Orthostatic Blood Pressure and pulse 2. Monitor Intake and output *Keep Urine Output over 30cc per hour 3. Monitor Hemoglobin or Hematocrit q1-2 hours prn 1. Keep Hemoglobin >10 g/dl or Hematocrit >30% 2. Packed Red Blood Cell transfusion as needed 4. Monitor coagulation studies 1. Fresh Frozen plasma transfusion as needed 2. Platelet transfusion as needed Answer: T, F(once pt stable), T, F, T

Page 68: Long case examination for phase iii medical students usmkk

In Pursuit to Excel MCQ Exam for Professional III Examination (MCQ)

11

The following are associated with placental insufficiency

A) Diabetes Mellitus B) Post maturity C) Twin pregnancy D) Cigarette smoking E) Dieting during pregnancy

Definition: Placental insufficiency is the failure of the placenta to supply nutrients to the fetus and remove toxic wastes. In post maturity of the baby [before term, growth of placenta is proportional to growth of fetus. However after term, placenta start to regress while the baby continue to grow] Causes [from Ten Teachers] Reduced uteroplacental perfusion: Inadequate trophoblast invasion, anti phospholipid syndrome, diabetes mellitus, Sickle cell disease, multiple gestation, collagen vascular disease. This will result in small placenta with gross morphological changes. Usually infarct and basal hematoma. Reduce feto placental perfusion: Single umbilical artery, twin-twin transfusion syndrome.

Antiphospholipid syndrome (APS) is a disorder that manifests clinically as recurrent venous or arterial thrombosis and/or fetal loss. Characteristic laboratory abnormalities in APS include persistently elevated levels of antibodies directed against membrane anionic phospholipids (ie, anticardiolipin [aCL] antibody, antiphosphatidylserine) or their associated plasma proteins, predominantly beta-2 glycoprotein I (apolipoprotein H); or evidence of a circulating anticoagulant. [EMedicine article 333221]

Twin-to-twin transfusion syndrome (TTTS) is the result of an intrauterine blood transfusion from one twin (donor) to another twin (recipient). TTTS only occurs in monozygotic (identical) twins with a monochorionic placenta. The donor twin is often smaller with a birth weight 20% less than the recipient's birth weight. The donor twin is often anemic and the recipient twin is often plethoric with hemoglobin differences greater than 5 g/dL. [EMedicine article 271752]

Answer: T, T, T, T, T

Page 69: Long case examination for phase iii medical students usmkk

In Pursuit to Excel MCQ Exam for Professional III Examination (MCQ)

12

Hydatidiform mole is associated with increased urinary output of

a) estrogen b) human chorionic gonadotrophin c) prostaglandin d) pregnanediol e) human placental lactogen

Gestational trophoblastic disease encompasses of several disease processes that Originate from placenta

a) Complete mole (no fetal tissue, 90% 46 XX and 10% 46 XY)

b) Partial mole (69, XXX or 69, XXY with fetal tissue present)

c) Placental site trophoblastic tumors d) Choriocarcinomas e) Invasive moles

Neutral steroids in urine were determined quantitatively with gaschromatography on capillary columns in a case of benign hydatidiform mole associated with bilateral theca-lutein cysts. A remarkable finding was the very high levels of 17-hydroxypregnanolone and pregnanetriol, which continued to rise until the 15th day after molar evacuation.[Vanluchene E et, al, 1977] Rapid urine qualitative hCG assays may not be reliable in the presence of markedly elevated hCG levels found in molar pregnancy. [Davison CM et al, 2004] Urinary pregnanediol levels, on the other hand are frequently decreased. [Clinical Laboratory Medicine by Richard Ravel] Decreased urinary pregnandeniol level but increased in serum progesterone and estradiol- 17 beta suggest molar pregnancy

Failure of the placental-site trophoblastic tumor to produce large amounts of estrogen, in contrast to normal pregnancy and Hydatidiform mole, resulted in marked androgen/estrogen imbalance, high circulating concentrations of free testosterone, and Virilization. [Nagelberrg SB& Rosen SW, 1985] A decrease of E3 in these abnormal pregnancies would result mainly in a lower level of tissue P450arom concentration [Takara Yamamoto et al, 1997] HPL Value increased in multiple pregnancies (twins or more), Placental site trophoblastic tumor, intact molar pregnancy, Diabetes, Rh incompatibility and deccreased in Toxemia, Aborting hydatidiform mole, Choriocarcinoma and Placental insufficiency [Medline Plus]

PGDH activity in neoplastic tissues was found to be one tenth or less of that in normal term human placentae. PGDH may be important in the accumulation of PGs in neoplastic tissues. If choriocarcinoma cells were able to synthesise prostaglandin E2 or PGF2α then the very limited amounts of PGDH in these cells would allow considerable quantities of these prostaglandins to be produced. The lack of PGDH in these cells suggests that very little or no PGE2 is synthesised in choriocarcinoma cells. Answer: T, T, F, F, T

Page 70: Long case examination for phase iii medical students usmkk

In Pursuit to Excel MCQ Exam for Professional III Examination (MCQ)

13

Prolactin a) is secreted by the posterior pituitary b) is necessary for mammary ductal

growth c) level in plasma is unaffected by

smoking d) is necessary for the establishment of

lactation e) secretion is controlled by an inhibiting

factor

Prolactin is a single-chain protein hormone closely related to growth hormone secreted by lactotrophs in the anterior pituitary. It is synthesized as a prohormone. Prolactin induces lobuloalveolar growth of the mammary gland. Alveoli are the clusters of cells in the mammary gland that actually secrete milk. Prolactin stimulates lactogenesis or milk production after giving birth. Prolactin, along with cortisol and insulin, act together to stimulate transcription of the genes that encode milk proteins.

Dopamine serves as the major prolactin-inhibiting factor or brake on prolactin secretion. It is secreted into portal blood by hypothalamic neurons, binds to receptors on lactotrophs, and inhibits both the synthesis and secretion of prolactin. Agents and drugs that interfere with dopamine secretion or receptor binding lead to enhanced secretion of prolactin. Prolactin secretion is positively regulated by several hormones, including thyroid-releasing hormone, gonadotropin-releasing hormone and vasoactive intestinal polypeptide. Stimulation of

the nipples and mammary gland, as occurs during nursing, leads to prolactin release. This effect appears to be due to a spinal reflex arc that causes release of prolactin-stimulating hormones from the hypothalamus. Estrogens provide a well-studied positive control over prolactin synthesis and secretion. Increase blood concentrations of estrogen during late pregnancy appear responsible for the elevated levels of prolactin that are necessary to prepare the mammary gland for lactation at the end of gestation. Excessive secretion of prolactin - hyperprolactinemia - is a relatively common disorder in humans. This condition has numerous causes, including prolactin-secreting tumors and therapy with certain drugs. Common manifestations of hyperprolactinemia in women include amenorrhea (lack of menstrural cycles) and galactorrhea (excessive or spontaneous secretion of milk). Men with hyperprolactinemia typically show hypogonadism, with decreased sex drive, decreased sperm production and impotence. Such men also often show breast enlargement (gynecomastia), but very rarely produce milk. http://www.vivo.colostate.edu/hbooks/pathphys/endocrine/hypopit/prolactin.html

A minimum of 5 cigarettes significantly decreases prolactin concentration in smokers. A matched pair’s comparison confirmed that smoking reduces the level of prolactin [Gabriela et al, 1995] Answer: F, T, F, T, T

Page 71: Long case examination for phase iii medical students usmkk

In Pursuit to Excel MCQ Exam for Professional III Examination (MCQ)

14

With regards of the blood volume and its composition during pregnancy

a) the total RBC falls by about 20% from the normal non-pregnant volume

b) the packed cell volume falls c) there is rise in the iron binding

capacity d) the blood cholesterol rises e) the protein bound iodine level falls

Physiology of anemia in pregnancy A disproportionate increase occurs in plasma volume (25% to 50%) compared to red blood cell mass (10% to 25%) during pregnancy, with resultant hemodilution and reduction in hematocrit of 3% to 5%. These changes begin at approximately 6 weeks gestation and normalize by 6 weeks postpartum. [The John Hopkins Manual of Gynecology and Obstetrics 3rd Ed.] Packed cell volume (PCV) is determined by measuring the height of the red cell volume in a micro-hematocrit capillary filled with whole blood, after centrifugation. It is a directly measured value, Meanwhile hematocrit (Hct) is the corresponding calculated value (RBC X MCV), PCV and Hct are interchangeable even though PCV is slightly higher than the more accurate Hct due to plasma trapping (between the packed cells in a centrifuged capillary). In present study hemoglobin percent and packed cell volume was significantly decreased in 2nd and 3rd trimester of pregnancy when compared with the control group and same category of women who were not supplemented with iron. It is evident that the significantly low hemoglobin percent and packed cell volume (PCV) in pregnant women is due in part to dietary iron deficiency. Therefore, iron therapy in pregnancy is helpful to maintain the hemoglobin percent and packed cell volume nearer to that of non pregnant normal women. [Wahed F et al] Total iron-binding capacity (TIBC) is a medical laboratory test which measures the blood's capacity to bind iron with transferrin.

[Wikipedia] In pregnancy, liver produces more transferrin. Serum Ferroxidase I and II are progressively increased with serum Total Iron Binding Capacity (TIBC) and unsaturated iron binding capacity (UIBC) as pregnancy advances [Agroyannis B et al] The total iron-binding capacity exceeded normal starting in the sixth lunar month in the nontreated group and in the seventh lunar month in the treated group, and it decreased slightly in the latter toward term and had returned to normal in both groups by the fourth postpartum week [George D. Malkasian] Serum protein bound iodine levels increased significantly with age in both sexes and in pregnant women, while it decreased significantly in alcoholics, cigarette smokers and marijuana addicts [O M Ebuehi et al] It is well known that with the effect of hormonal changes during pregnancy, plasma lipid levels increase. Expected elevations for triglyceride and cholesterol levels during a normal gestational period usually do not exceed 332 mg/dL and 337 mg/dL, respectively (corresponding 95th percentile values). However, elevations over the 95th percentile values can be observed during pregnancy, and patients with levels over these expected adaptation levels can be divided into 2 groups: (1) supraphysiologic hyperlipoproteinemia during pregnancy and (2) extreme hyperlipoproteinemia limited to gestational period (triglyceride level >1000 mg/dL) [Basaran A.] Answer: F, T, T, T, F

Page 72: Long case examination for phase iii medical students usmkk

In Pursuit to Excel MCQ Exam for Professional III Examination (MCQ)

15

Ovulation in human

a) is associated with surge of LH b) is characteristically followed by the

development of secretory endometrium

c) is associated with an increased in motility of the Fallopian tube

d) is associated with a sustained fall in the basal body temperature

e) followed by a rise in urinary pregnanetriol

The midcycle LH surge is responsible for a dramatic increase in local concentrations of prostaglandins and proteolytic enzymes in the follicular wall. These substances progressively weaken the follicular wall and ultimately allow a perforation to form. Ovulation most likely represents a slow extrusion of the oocyte through this opening in the follicle rather than a rupture of the follicular structure [Novak Gynecology 14 ed]

Peristaltic movement of fallopian tube is primarily regulated by three intrinsic systems: the estrogen-progesterone hormonal milieu, the adrenergic-nonadrenergic system, and prostaglandins. Before ovulation, contractions are gentle, with some individual variations in rate and pattern. At ovulation, contractions

become vigorous and the mesosalpinx contracts to bring the tube in more contact with the ovary while the fimbria contracts rhythmically to sweep over the ovarian surface. As progesterone level rises 4-6 days after ovulation, it inhibits tubal motility. This may lead to relaxation of the tubal musculature to allow passage of the ovum into the uterus by the action of the tubal cilia. The effects of estrogen and progesterone on oviductal motility and morphology is mediated through these steroids' receptors. The changes in receptors levels are critical in determining the functional state of the oviduct.[Diaa M. EI-Mowafi, MD;Zagagig University, Egypt] In women, ovulation causes an increase of one-half to one degree Fahrenheit (one-quarter to one-half degree Celsius) in basal body temperature (BBT); monitoring of BBTs is one way of estimating the day of ovulation. The tendency of a woman to have lower temperatures before ovulation, and higher temperatures afterwards, is known as a biphasic pattern. Charting of this pattern may be used as a component of fertility awareness.[Wikipedia] During the normal menstrual cycle the excretion of pregnanetriol increased on the day that the excretion of oestrone and oestradiol reached a maximum during the follicular phase. Pregnanediol excretion did not begin to increase until 2 days later. The excretion of pregnanetriol reached a peak and began to decrease before that of pregnanediol. Thus the pattern of pregnanetriol excretion was different from that of the excretion of oestrogens or pregnanediol. [K. Fotherby] It is found that the cyclic change in urinary pregnanetriol excretion is variable in extent but there is a correlation between the urinary excretion of oestrogens, pregnanediol and pregnanetriol during the ovulatory cycle [Mary TP & Ian FS] Answer: T, T, T, F, T

Page 73: Long case examination for phase iii medical students usmkk

In Pursuit to Excel MCQ Exam for Professional III Examination (MCQ)

16

Pre eclampsia is associated with A) Diabetes Mellitus B) Rhesus iso immunization C) Urinary tract infection D) Polyhydramnios E) Twin pregnancy

Severe pre-eclampsia and eclampsia are relatively rare but serious complications of pregnancy, with around 5/1000 maternities in the UK suffering severe pre-eclampsia and 5/10 000 maternities suffering eclampsia. In eclampsia, the case fatality rate has been reported as 1.8% and a further 35% of women experience a major complication [RCOG Guideline No. 10(A)] Who is at risk of getting pre-eclampsia?

1) First pregnancy 2) First pregnancy with a new partner 3) Aged 40 or over 4) Mother or sister had pre-eclampsia

during pregnancy 5) Pre-eclampsia in a previous pregnancy 6) Body mass index (BMI) of 35 or more

(you weigh 90 kg or more) 7) Multiple pregnancy 8) Medical problem such as high blood

pressure, kidney problems and/or diabetes.

9) Pregnant from egg (oocyte) donation. Controlled cohort studies showed that the risk of pre-eclampsia is increased in women with a previous history of pre-eclampsia (relative risk 7.19, 95% confidence interval 5.85 to 8.83) and in those with antiphospholipids antibodies (9.72, 4.34 to 21.75), pre-existing diabetes (3.56, 2.54 to 4.99), multiple (twin) pregnancy (2.93, 2.04 to 4.21), nulliparity (2.91, 1.28 to 6.61), family history (2.90, 1.70 to 4.93), raised blood pressure (diastolic ≥ 80 mm Hg) at booking (1.38, 1.01 to 1.87), raised body mass index before pregnancy (2.47, 1.66 to 3.67) or at

booking (1.55, 1.28 to 1.88), or maternal age ≥ 40 (1.96, 1.34 to 2.87, for multiparous women). Individual studies show that risk is also increased with an interval of 10 years or more since a previous pregnancy, autoimmune disease, renal disease, and chronic hypertension. [Duckitt & Harrington] The most significant risk factors for developing pre-eclampsia are a history of pre-eclampsia and the presence of Antiphospholipid antibodies. [Duckitt & Harrington] Pre-existing diabetes and a pre-pregnancy BMI of ≥ 35 almost quadruple the risk; nulliparity, a family history of pre-eclampsia, and twin pregnancy almost triple the risk; and maternal age ≥ 40, a booking BMI of ≥ 35, and a systolic blood pressure ≥ 130 at booking double the risk. [Duckitt & Harrington]

Pre-existing hypertension, renal disease, chronic autoimmune disease, and ≥ 10 years between pregnancies increase the risk but it is not clear by how much. [Duckitt & Harrington] These data demonstrates a significant positive relation with maternal age, diabetes in pregnancy, and fetal macrosomia with polyhydramnios. Anemia during pregnancy, cesarean delivery rate, and congenital anomalies were significantly higher in the study group.[Mathew Mariam et al]

Answer: T, F, F, F, T

Page 74: Long case examination for phase iii medical students usmkk

In Pursuit to Excel MCQ Exam for Professional III Examination (MCQ)

17

In normal labour

a) endogenous oxytocin is responsible for the initiation of uterine contraction

b) there is progressive increase in normal cortisol

c) the uterine contractions is increased by release of endogenous prostaglandin

d) during the first stage of labour, the maternal arterial pressure rises during each uterine contraction

They are four major hormonal systems involve during labor which are oxytocin, endorphins, epinephrine& Norepinephrine and prolactin. Oxytocin causes the rhythmic uterine contractions of labor, and levels peak at birth through stimulation of stretch receptors in a woman’s lower vagina as the baby descends. The high levels continue after birth, culminating with the birth of the placenta, and then gradually subside. The baby also has been producing oxytocin during labor, perhaps even initiating labor; [Dr Sarah J Buckley MD] These results give further support to the hypothesis that maternal stress leads to a reduced concentration of prolactin and increased concentration of cortisol whereas relief of pain and maternal anxiety with meperidine lessens both effects.[E. Onur et al] Maternal plasma Cortisol levels in 62 primiparous women rose during labour and remained high for 20 minutes after delivery. Umbilical cord Cortisol levels were substantially lower than maternal levels. The maternal Cortisol level was slightly lower in those who required oxytocin. This relationship between maternal Cortisol levels and the use of oxytocin was not affected by the use of epidural analgesia. The mean maternal Cortisol level rose to a lesser extent in the women who had epidural analgesia than in those who did not.[Haddad & Morris]

It is widely accepted that prostaglandins play a central role in parturition through their actions on uterine contractility and on cervical ripening. In both human and nonhuman primates, the major intrauterine sources of prostaglandins are the fetal membranes and the deciduas [George J. Haluska et al]. PGs have also been shown to play an important role in up-regulation of the fetal hypothalamic-pituitary-adrenal axis, membrane rupture and the maintenance of uterine and placental blood flow [McKeon & Challis] 13, 14-dihydro-15-keto-prostaglandin F (PGFM) levels increased a labor progressed, and reached maximal levels before placental separation had occurred. Peripheral plasma concentrations of oxytocin did not change significantly at any stage of labor or 2 hours post partum. These results suggest that prostaglandins have a role in the third stage of labor, and this finding may have important clinical implications. [SM Sellers et al] With active labor there are clear identifiable changes in arterial compliance and cardiac load as reflected in rapid ejection times (RETs) and pulse wave arrival times (PWATs), respectively. These findings are absent in Prelabour. [Edward H. Hon et al] Serial measurements of cardiac output and mean arterial pressure were performed in 15 women during the first stage of labour and at one and 24 hours after delivery.....Over the same period basal mean arterial pressure also increased......There were also further increases in mean blood pressure during contractions.[S C Robson et al] Answer: T, T, T, T, T

Page 75: Long case examination for phase iii medical students usmkk

In Pursuit to Excel MCQ Exam for Professional III Examination (MCQ)

18

Regarding ectopic pregnancy

a) the common site is the cornu of uterus b) anaemia is due to thrombocytopenia c) hemoperitoneum results from tubal

rupture d) ovary is another common site e) shock may follow tubal rupture

Ectopic pregnancy refers to the implantation of a fertilized egg in a location outside of the uterine cavity, including the fallopian tubes, cervix, ovary, cornual region of the uterus, and the abdominal cavity. This abnormally implanted gestation grows and draws its blood supply from the site of abnormal implantation. As the gestation enlarges, it creates the potential for organ rupture because only the uterine cavity is designed to expand and accommodate fetal development. Ectopic pregnancy can lead to massive hemorrhage, infertility, or death. [Vicken P Sepilian] It is occur in 2% of all pregnancy

Anything that impairs migration of embryo to the andometrial cavity will predispose a woman to ectopic pregnancy. this includes pelvic inflammatory disease, history of prior ectopic pregnancy, History of tubal surgery and conception after tubal ligation, Use of fertility drugs or assisted reproductive technology, Use of an intrauterine device, Increasing age, smoking, Salpingitis isthmica nodosum, diethylstilbestrol (DES) exposure, a T-shaped uterus, prior abdominal surgery, failure with progestin-only contraception, and ruptured appendix.

Sites and frequencies of ectopic pregnancy. By Donna M. Peretin, RN. (A) Ampullary, 80%; (B) Isthmic, 12%; (C) Fimbrial, 5%; (D) Cornual/Interstitial, 2%; (E) Abdominal, 1.4%; (F) Ovarian, 0.2%; (G) Cervical, 0.2%.

Pictures from http://emedicine.medscape.com/article/258768-overview

The classic clinical triad of ectopic pregnancy is pain, amenorrhea, and vaginal bleeding (50%). Patients may present with other symptoms common to early pregnancy, including nausea, breast fullness, fatigue, low abdominal pain, heavy cramping, shoulder pain, and recent dyspareunia.

38 year-old woman presented with gynaecologic haemorrhage, pelvic pain and hypovolemic shock. Without any ambiguity, the diagnosis was directly made during contrast enhanced Multidetector Computed Tomography (MDCT). Massive hemoperitoneum with fresh blood clots in the hypogastric area, active free peritoneal extravasation of intravascular contrast material and dramatic peripheral enhancement, - "ring of fire" sign - of an adnexal cystic structure were the key signs. [Coulier B et al]

Finding of hemoperitoneum on ultrasound examination may not be an absolute contraindication to conservative management of tubal ectopic pregnancy [Bignardi & Condous]

Rupture may be heralded by sudden, severe pain, followed by syncope or by symptoms and signs of hemorrhagic shock or peritonitis. Rapid hemorrhage is more likely in ruptured cornual pregnancies [The Merck Manuals]

Answer: F, F, T, F, T

Page 76: Long case examination for phase iii medical students usmkk

In Pursuit to Excel MCQ Exam for Professional III Examination (MCQ)

19

Endometriosis

a) commonly affects the ovaries b) is usually not associated with

infertility c) intestinal obstruction is a possible

complication if bowels are involved d) often present with dysmenorrhea e) is primarily an acute inflammatory

process

Endometriosis is the presence of endometrial-like tissue outside the uterine cavity, which induces a chronic inflammatory reaction. It can occur in various pelvic sites such as on the ovaries, fallopian tubes, vagina, cervix, or uterosacral ligaments or in the rectovaginal septum. It can also occur in distant sites including laparotomy scars, pleura, lung, diaphragm, kidney, spleen, gallbladder, nasal mucosa, spinal canal, stomach, and breast. [Ami K Davé, MD]

Ovaries is the most common site for implantation of the endometrial cell (up to 75%) because 1) through the theory of retrograde menstruation, ovaries are adjacent to the opening of the tube in the pelvic area and that location alone will make the ovaries more prone to be contaminated with the regurgitated menstrual flow.2) ovaries have the highest level of steroid hormone compared to any other organ and hence they represent an ideal environment for implantation and growth of the endometrial tissue.

Infertility in endometriosis are due to 1) increase level of prostaglandin production which is interfering with sperm-ovum interaction, embryo growth, interfering with sperm motility, and interfering with the function of some central nervous system areas that are responsible for control of reproduction and 2) The presence of endometrial tissue in the pelvic and peritoneal cavities will cause some degree of abnormality

or inadequacy of ovulation. Some studies say that up to 50 percent of patients with endometriosis have some degree of ovulatory dysfunction and this should certainly be taken into account in the treatment of patients with endometriosis and infertility. [Endometriosis - The '90s Outlook]

Usually, the pain associated with endometriosis is right before or during the menstrual period in the initial stages; however, as the disease progresses, it may occur throughout the cycle. The pain may be acute or chronic. In about half of the patients with severe or extensive endometriosis, the pain is chronic all through the cycle which gets worse right before and during menstruation, and during or shortly after intercourse. [Endometriosis - The '90s Outlook]

Endometriosis may invade the rectovaginal septum and the anterior rectal wall. It may also involve the upper rectum and sigmoid colon, infiltrating the muscularis. Cyclical rectal bleeding (hematochezia) is pathognomonic of endometriosis. However, transmural bowel involvement by endometriosis remains a rarity. The ileum, appendix, and cecum may also be involved, leading to intestinal obstruction. Cicatrization as a consequence of endometriosis may lead to symptoms of obstruction even in postmenopausal women. [Ami K Davé, MD]

Answer: T, F, T, T, F

Page 77: Long case examination for phase iii medical students usmkk

In Pursuit to Excel MCQ Exam for Professional III Examination (MCQ)

20

Question1: Which of the following statements are true regarding the pharmacological induction of fetal respiratory maturity? a) Maternally administered thyroid releasing hormone (TRH) alone may cause lung maturation. b) Antenatal TRH has no maternal side-effects. c) Treatment with antenatal corticosteriods between 28 and 32 weeks reduces the incidence of fetal intraventricular haemorrhage. d) The use of antenatal cortiosteriods between 28 and 32 weeks with pre-labour ruptured membranes is associated with a significant increase in neonatal infection. e) Antenatal corticosteriods may cause long-term cognitive disabilities in infants treated prenatally. a) False b) False c) True d) False e) False The meta-analysis of 36 prospective studies indicate that treatment between 28 and 32 weeks with antenatal corticosteriods prior to the onset of premature labour resulted in a substantial reduction in the incidence of respiratory distress syndrome. The reduction in respiratory morbidity was associated with overall reductions in the incidence of neonatal intraventricular haemorrhage, necrotizing enterocolitis and early neonatal death. There was no strong evidence of any adverseeffects of corticosteroids in these trials. Long-term follow-up of children in several of these studies indicated no adverse effect on growth, physical development or cognitive skills. Trials investigating the improved efficacy of antenatal TRH given with antenatal corticosteriods, as compared to corticosteriods

alone have been described. The Cochrane Database indicates that with the available evidence, antenatal TRH cannot be recommended for clinical practice at the present time. Adverse maternal side-effects were significant for women receiving antenatal TRH and these included hypertensive episodes. Q11: Beta sympathomimetic drugs: a) Increase maternal stroke volume. b) Decrease fetal heart rate. c) Increase maternal bowel motility. d) Decrease intracellular potassium in maternal cells. e) Decrease maternal urinary output. Answer: T, F,F,F,T Beta adrenergic agonists, currently the most widely used tocolytic drugs in the UK, include ritodrine, salbutamol and terbutaline. These pharmacological agents act via Beta-1 and Beta-2 adrenergic receptors. Beta-I receptor responses include an increase in maternal heart rate and stroke volume, a decrease in bowel motility and an increase in metabolic lipolysis. The Beta-2 adrenergic response includes an increase in renal renin production and a decrease in urinary output. Question, answer and discussion are taken from Self-assessment questions: Premature labour, by M.Kilby, Current Obstetrics & Gynaecology (1996) volume 6 issue 3

Page 78: Long case examination for phase iii medical students usmkk

In Pursuit to Excel MCQ Exam for Professional III Examination (MCQ)

21

Question 2: The following are contraindications to tocolytic inhibition of preterm labour: a) Abruptio-placenta. b) Chorioamnionitis. c) Pre-labour ruptured membranes. d) Severe pregnancy induced hypertension. e) Mild intrauterine growth restriction. a) True b) True c) False d) True e) False Antepartum haemorrhage of whatever cause is a contraindication to the suppression of preterm labour as is intrauterine infection and severe pregnancy induced hypertension which may be life threatening to the mother. In prelabour rupture of membranes, although there is no absolute contraindication to tocolysis as long as there is no overt evidence of chorioamnionitis,there is little evidence to suggest that tocolysis significantly improves perinatal mortality. In selected cases of mild intrauterine growth restriction in which the fetus is not compromised, it may be beneficial to suppress labour to allow antenatal corticosteriods to be administered. However, any evidence of fetal compromise should lead to prompt delivery of the fetus. Question3: During a preterm delivery: a) Epidural anaesthesia is contraindicated if chorioamnionitis is present. b) Elective episiotomy is advised for all vaginal deliveries. c) The elective use of low outlet forceps delivery

significantly reduces intraventricular haemorrhage. d) Antibiotic administration significantly reduces maternal infective morbidity in prelabour ruptured membranes. e) Intrapartum antibiotic therapy significantly reduces fetal infective morbidity in prelabour ruptured membranes. a) True b) False c) False d) True e) True Analgesia and anaesthesia of choice for preterm labour and delivery are not well established. Epidural anaesthesia may well have benefits especially in the management of the first and second stage of a preterm vaginal breech delivery. However, such a choice of analgesia is contraindicated in chorioamnionitis. Although both an elective episiotomy and low forceps delivery have been historically discussed as useful in reducing the length of second stage and protecting the premature fetus at delivery, there are no data to support this as a potential benefit to the fetus. Administration of antibiotics prophylactically to women with prelabour ruptured membranes preterm delays delivery and reduces both maternal and neonatal infection. No effect has yet been shown demonstrating an improvement in perinatal mortality. In particular, in women known to be carrying Group B Streptococci, intrapartum antibiotics should be adopted as standard care. Question, answer and discussion are taken from Self-assessment questions: Premature labour, by M.Kilby, Current Obstetrics & Gynaecology (1996) volume 6 issue 3

Page 79: Long case examination for phase iii medical students usmkk

In Pursuit to Excel MCQ Exam for Professional III Examination (MCQ)

22

Q4: The following drugs significantly reduce preterm labour when administered: a) Ritodrine. b) Indomethacin. c) Nifedipine. d) Pethidine. e) Dexamethasone. ANSWER 4 a) True b) False c) False d) False e) False Although beta-sympatho-mimetics themselves may not have beneficial effects for the fetus they are effective at postponing delivery, especially for short intervals of up to 24 h. This suggests that they may have a place if, for instance, corticosteriods could be administered to promote pulmonary maturity. However, injudicious use of corticosteriods and beta-sympathomimetics have been reported to precipitate acute pulmonary oedema. The only statistically significant differences in prospective trials comparing the use of calcium antagonists with beta-sympatho-mimetics have indicated that there were fewer neonates of less than 2.5 kg and there were more admissions to neonatal' intensive care after the treatment with calcium antagonists. At the present time, only a small number of prospective studies are reported. Meta-analysis from the Cochrane Database does not support the use of calcium antagonists or magnesium sulphate in preference to beta-sympatho-mimetics in the suppression of preterm labour. Pethidine may suppress both fetal and maternal respiration and has no proven benefit in the management of preterm labour.

There is no evidence that corticosteriod administration suppresses preterm labour and there is anecdotal evidence indicating that intra-amniotic injection of betamethasone may actually be used to induce labour. Q5: The following are useful predictors of preterm labour: a) A previous history of preterm labour. b) Maternal plasma oestradiol concentration 14 days prior to onset of labour. c) Fetal fibronectin. d) Multiple pregnancy. e) Uterine anomaly. Answer: T, F, T, T, T The background rate of preterm labour in the UK is approximately 7%. If a patient has had a previous preterm labour this rises to between 14 and 18%. A uterine anomaly is proven in between 5 and 16% of all preterm deliveries. Multiple pregnancy is associated with preterm delivery and this is paticularly true with monochorionic placentation. The majority of biochemical prediction indices, including serum oestradiol concentrations, have a poor predictive index at detecting preterm delivery. Fetal fibronectin, a component of the extra cellular matrix in the cervix, has been shown both in cross sectional studies, and more recently in longitudinal studies, to have a positive predictive value of preterm delivery in high risk patients of 46%. Question, answer and discussion are taken from Self-assessment questions: Premature labour, by M.Kilby, Current Obstetrics & Gynaecology (1996) volume 6 issue 3

Page 80: Long case examination for phase iii medical students usmkk

In Pursuit to Excel MCQ Exam for Professional III Examination (MCQ)

23

Q6: Regarding the lecithin-sphingomyelin ratio: a) This test is carried out on amniotic fluid. b) Sphingomyelin is a general membrane lipid. c) The ratio for normal pregnancies is greater than 0.5 at 20 weeks. d) If the ratio is less than 2 there is less than a 10% chance of the baby developing respiratory distress syndrome. e) Lecithin is produced by type II pneumocytes. Answer: T, T, F, F, T The L/S ratio was introduced by Gluck in 1971. Amniocentesis allows the collection of fluid and the results are expressed as the ratio of lecithin (phosphatylcholine) fraction enriched by cold acetone precipitation. The sphingomyelin is used as a control because amniotic fluid volume changes during gestation cannot be accurately measured clinically. In normal pregnancies the L/S ratio is less than 0.5 at 20 weeks. The value of 2.0 indicates a low risk of respiratory distress syndrome at any point in gestation. The L/S ratio is not reliable if amniotic fluid is heavily contaminated with blood or meconium. Lecithin and other phosphatyl-lipids are produced by the type II pneumocytes within the lungs. Q7: With cervical incompetence: a) In-utero exposure to Diethylstilboestrol (DES) is a causative factor. b) A history of loop diathermy excision of the transformation zone of the cervix for treatment of CIN is a common association. c) Shirodkar and McDonald cerclage techniques carry the same success rates. d) When the amniotic membranes are ruptured then cervical cerclage is contraindicated. e) This may be treated by oral Ritodrine. ANSWER 7 a) True b) False

c) True d) True e) False Cervical incompetence is a dysfunction in the integrity ofthe internal cervical os. It is characterised by painless cervical dilatation in the mid second trimester. With regard to aetiology, several factors are important. In-utero exposure of Diethylstilboestrol carries a 45% risk of pregnancy loss due to cervical incompetence. Acquired factors include cervical trauma. This includes over zealous mechanical dilatation of the cervix prior to diagnostic curettage. Loop excision of the transformation zone is rarely associated with cervical incompetence. With contemporary cervical cerclage, the majority of patients have their operation during pregnancy. The classical Shirodkar's suture and McDonald's suture have approximately the same success rate in prevention of premature labour. Ritodrine alone will not stop the contractions due to cervical incompetence and there is little prospective evidence to confirm that ritodrine therapy has a role to play perioperatively in cervical cerclage. Question, answer and discussion are taken from Self-assessment questions: Premature labour, by M.Kilby, Current Obstetrics & Gynaecology (1996) volume 6 issue 3

Page 81: Long case examination for phase iii medical students usmkk

In Pursuit to Excel MCQ Exam for Professional III Examination (MCQ)

24

QUESTION 8: In preterm labour complicated by a low-grade infection: a) Bacterial phospholipases cause a direct release of arachidonic acid from the amnion and decidual cells. b) Bacterial lipopolysaccharides stimulate increased production of interleukin-I (1L-I) from amnion and decidual cells. c) IL-1 stimulates the decidual production of IL8 d) Corticotrophin releasing hormone (CRH) does not stimulate prostaglandin production from decidual cells. e) Annexin production by the trophoblast and decidua decrease with the onset of labour. ANSWER: T, T, T, F, T Approximately 30% of all preterm labours are associated with some mild chorioamnionitis. Bacterial phospholipases directly release arachidonic acid which are precusors of many eieosanoids from decidual cells. Also lipopolysaccharides stimulate decidual cells to increase cytokines such as IL-I and tumour necrosis factor from amnion and decidual cells. The cytokines are elevated in the amniotic fluid from patients with preterm labour and infection and may secondarily release other cytokines such as IL-8 which are chemotaxtic peptides for neutrophils and T-cells. Cortieotrophin releasing hormone is produced by the syncytio-trophoblast and stimulates prostaglandin production from cultured decidual cells. The prostaglandins produced may have a direct effect upon the myometrium which is adjacent to the chorion, amnion and decidual cells.The annexins are substrates for tyrosine kinases and influence phospholipase A2 production and, therefore, may increase myometrial intracellular free calcium concentration. It is believed that they bind to phospholipids preventing phospholipase A2 from gaining access to its substrate. Both annexin protein and mRNA are decreased in the

syncytio-trophoblast of patients with preterm labour. Q9: Relaxin: a) Has a stimulatory effect on the myometrium. b) Has an inhibitory effect that is more rapid than progesterone. c) Affects frequency modulation of uterine contractions. d) Elevates uterine cyclic AMP concentrations. e) Stimulates prostocyclin production by the myometrium. Answer: F, T, T, T, T Experimental animal models have suggested that Relaxin has an inhibitory effect on myometrial activity. This is separate from that induced by progesterone. The inhibitory effect of Relaxin is more rapid in onset than that of progesterone. Although spontaneous myometrial contractility is suppressed, sensitivity to oxytocin is maintained and its primary effects are one of frequency modulation of contractions. Relaxin also elevates uterine cyclic AMP production in common with other uterine relaxants. It is unknown whether Relaxin has its effect directly on the myometrium, but experimental evidence indicates that cultured myometriai cells, when stimulated with Relaxin, produce prostacyclin. Question, answer and discussion are taken from Self-assessment questions: Premature labour, by M.Kilby, Current Obstetrics & Gynaecology (1996) volume 6 issue 3

Page 82: Long case examination for phase iii medical students usmkk

In Pursuit to Excel MCQ Exam for Professional III Examination (MCQ)

25

Q7: which of the following statements concerning placental abruption is/are true: a) In western society placental abruption is commonly associated with poor nutrition. b) Placental abruption can occur in association with external cephalic version. c) Placental abruption is more common with multiple pregnancy. d) Placental abruption is more common with a history of previous abruption. e) Postpartum haemorrhage is a common complication of placental abruption, which often requires hysterectomy to control the bleeding. a) False. Poor nutrition has previously been reported as being associated with placental abruption, but within the western world this is now a rare finding. b) True. External cephalic version is associated with placental abruption, especially if the manoeuvre is carried out under general anaesthetic. Presumably, if the manoeuvre is carried out under general anaesthetic, the operator exerts more 'energy' into the procedure as she/he does not have the benefit of appreciating the maternal perception of discomfort. c) True. d) True. e) False. Postpartum haemorrhage is a common sequel of placental abruption, but only rarely is hysterectomy required for its control. Senior obstetric input is mandatory in the management of postpartum haemorrhage under these circumstances, as it is of the utmost importance that a hysterectomy is performed neither too early nor too late.

Q5: Concerning placenta praevia, which of the following statements is/are true: a) The incidence of placenta praevia is 1 in 5000. b) Placenta praevia is more common in Caucasian women. c) Owing to the position of the placenta in placenta praevia, prolapsed cord is much less likely. d) Previous lower segment Caesarean section is a risk factor for placenta praevia. e) There is no association between the incidence of placenta praevia and maternal age. a) False. The true incidence is 0.3-1.0%. b) False. Placenta praevia is more common in black women. c) False. Prolapsed cord is up to three times more likely with placenta praevia. d) True. e) False. More common in older women and multiparous women. Question, answer and discussion are taken from Self-assessment questions: The Placenta, by S.Smith, Current Obstetrics & Gynaecology (1998) volume 8 issue 1

Page 83: Long case examination for phase iii medical students usmkk

In Pursuit to Excel MCQ Exam for Professional III Examination (MCQ)

26

Question 4: Anti phospholipid syndrome: a) The mechanism of action of antiphospholipid antibodies is unknown. b) In the presence of antiphospholipid antibodies, pregnancy losses occur with equal frequency in all three trimesters. c) The treatment of choice is with steroids and aspirin. d) Anticardiolipin antibody is a very specific marker of pregnancy loss. Answer: T, F, F, F, T Various pathways have been suggested to explain the action of antiphospholipid antibodies: coagulation in small vessels leading to the placenta, vasoconstriction owing to inhibition of prostacycline, and disruption of the phospholipid intracellular bridges between elements of trophoblast. In the presence of antiphospholipid antibodies, approximately 60% of pregnancy losses will be in the first trimester, but there is a heavy preponderance (40%) of second- and third-trimester losses. Various treatment regimens have been used. At present heparin (or low-molecular-weight heparin) and low-dose aspirin seems to be the most effective regimen. Steroids are no longer widely used owing to the possible side effects. However, the issue is far from settled. Anticardiolipin antibody is a very non-specific marker of pregnancy loss. It can be raised after viral infections etc. It is only of real significance if a high level is present, or if thromboses and/or other autoantibodies are present. Treatment of thrombophilias, has mainly been found to be effective in cases of late pregnancy loss rather in first trimester miscarriage.

Question, answer and discussion are taken from Self-assessment questions: Recurrent Abortion by H.J.A. Carp, Current Obstetrics & Gynaecology (1999) volume 9 issue 1

Page 84: Long case examination for phase iii medical students usmkk

In Pursuit to Excel MCQ Exam for Professional III Examination (MCQ)

27

Question 1: Regarding the diagnosis and classification of hypertension in pregnancy: a) The phase 5 Korotkoff sound is more reliably detected in pregnancy than the phase 4 sound. b) ‘Significant proteinuria’ is denoted by the presence of 500 mg of protein in a 24 h collection. c) The combination of hypertension and proteinuria always signifies the presence of preeclampsia. d) The presence of oedema is a useful diagnostic sign. e) Preeclampsia never presents before 20 weeks’ gestation. Answer: T, F, F, F, F Pre-eclampsia is generally defined as hypertension of at least 140/90 mmHg measured on at least two separate occasions and arising de novo after the 20 weeks’ gestation, in the presence of 300 mg of protein in a 24 h collection of urine.While the rapid appearance of oedema (particularly when it involves the face) may herald the advent of pre-eclampsia, this sign is not part of the diagnostic pattern and, indeed, is present in two-thirds of normal pregnant women. The combination of hypertension and proteinuria does not always signify preeclampsia. Renal disease often presents with proteinuria and may or may not be accompanied by hypertension. This is the more likely diagnosis if the presentation is before 20 weeks’ gestation. However, a hydatidiform mole can rarely present in the first trimester with preeclampsia. The measurement of blood pressure in pregnancy has long been the subject of much controversy. The evidence at present suggests that Korotkoff phase V sound is the most reproducible endpoint in pregnancy.

Question 2: Concerning the epidemiology of preeclampsia: a) The incidence of preeclampsia in primigravid mothers in the UK is 1%. b) Preeclampsia is more common in the black population. c) The incidence of preeclampsia is higher in triploid pregnancy. d) The incidence of preeclampsia is higher in lower socio-economic groups. e) The risk of developing preeclampsia is decreased in a pregnancy complicated by placenta praevia. Answer: F, T, T, T, T The incidence of pre-eclampsia in privigravid women in the UK is between 7 and 10%. Some ethnic groups appear to be at increased risk and in particular the black population is said to have an increased risk. It is, however, difficult to separate this risk from other predisposing factors such as parity, obesity and an inherited tendency to essential hypertension. There is a well-documented relationship between hyperplacentosis and the development of pre-eclampsia; thus, multiple, molar and triploid pregnancy are all associated with an increased incidence of pre-eclampsia. Placenta praevia is traditionally said to confer a decreased risk of developing pre-eclampsia and certainly the incidence of one is lower in the presence of the other. However, the protective effect of placenta praevia may merely reflect the increased incidence of earlier delivery associated with this condition. A higher incidence of pre-eclampsia has been reported in lower socio-economic groups. This difference most likely reflects differences in age, parity, levels of antenatal care and smoking. Question, answer and discussion are taken from Self-assessment questions: The pathogenesis of pre-eclampsia, by L.C. Kenny, Current Obstetrics & Gynaecology (1999) volume 9 issue 4

Page 85: Long case examination for phase iii medical students usmkk

In Pursuit to Excel MCQ Exam for Professional III Examination (MCQ)

28

Question 3: Pre-eclampsia is associated with: a) An increase in the maternal platelet count. b) An increase in the maternal mean platelet volume. c) A decrease in maternal thromboxane production. d) An increase in circulating maternal levels of b- thromboglobulin. e) An increase in platelet adhesion. Answer: F, T, T, T, T Longitudinal analysis of the peripheral platelet count in pregnancy has revealed that pre-eclampsia is associated with thrombocytopaenia. As a result the mean platelet volume has been reported to increase in preeclampsia. The population of larger platelets in this condition may be explained by increased consumption, leading to an increase in the proportion of immature, larger platelets in the peripheral circulation. There is substantial evidence of platelet activation in pre-eclampsia. Circulating levels of factors stored within platelets reflect platelet activation Several studies have reported increased levels of the platelet granule protein b-thromboglobulin in women with pre-eclampsia as compared with normal pregnant controls and this elevation precedes the development of clinical signs by at least 4 weeks. Thromboxane production, as measured by urinary metabolites, is increased in women with pre-eclampsia. This increase in thromboxane production leads to an increase in platelet adhesion and aggregation . Question 4: Pre-eclampsia is associated with: a) Increased maternal serum concentrations of highdensity lipoprotein cholesterol. b) Decreased maternal concentrations of serum triglycerides. c) Increased maternal serum homocysteine concentrations.

d) Decreased maternal levels of vitamin E. e) An increase in the maternal ratio of prostacyclin to lipid peroxides. Answer: F, F, T, T, F There are many emerging similarities between the condition of atherosclerosis and pre-eclampsia. The two conditions share a similar lipid profile; low maternal maternal serum concentrations of HDL cholesterol, raised concentrations of serum triglycerides, and increased formation of small, dense LDL particles. Furthermore, many of the risk factors for the two disorders are similar; obesity, black race, lipid abnormalities, insulin resistance and raised homocysteine concentrations all predispose to atherosclerosis and pre-eclampsia. These similarities and the generally accepted role of oxidative stress in atherosclerosis, support the emerging concept that reduced placental perfusion interacts with maternal factors to generate oxidative stress in pre-eclampsia. The most important lipid-soluble anti-oxidant in human plasma is vitamin E. In normal pregnancy plasma levels of prostacyclin and vitamin E increase, whereas thromboxane levels are decreased and serum levels of lipid peroxide remain relatively constant. In pre-eclampsia, there is an imbalance in the thromboxane to prostacyclin ratio with elevated maternal levels of lipid peroxides and decreased levels of vitamin E. Question, answer and discussion are taken from Self-assessment questions: The pathogenesis of pre-eclampsia, by L.C. Kenny, Current Obstetrics & Gynaecology (1999) volume 9 issue 4

Page 86: Long case examination for phase iii medical students usmkk

In Pursuit to Excel MCQ Exam for Professional III Examination (MCQ)

29

Question 5: Maternal cardiovascular changes associated with preeclampsia include: a) An increase in maternal plasma volume. b) A decrease in the maternal haematocrit. c) A rise in the maternal plasma oncotic pressure. d) An exaggerated maternal arterial response to angiotensin II. e) An exaggerated maternal arterial response to bradykinin. Answer: F, F, F, T, F In normal pregnancy, plasma volume increases by about 40% and leads to a fall in the haematocrit. The traditional and most widely accepted model of the haemodynamic characteristics of pre-eclampsia is one of a relatively reduced plasma volume, vasoconstriction and resulting hypoperfusion of organs such as the placenta and kidneys. Thus, the haematocrit in pre-eclampsia is typically increased. The reduced plasma volume associated with pre-eclampsia reflects the shift of fluid into the extravascular space across leaky capillary membranes. Increasing proteinuria in pre-eclampsia leads to marked hypoalbuminaemia and a fall in plasma oncotic pressure. In normal pregnancy, a relative arterial refractoriness to angiotensin II develops. This can be detected as early as 8 weeks and is maximal at term. In women who develop pre-eclampsia, the pressor response to angiotensin II remains relatively greater than in normal women. Ex vivo studies have demonstrated that endothelium- dependent relaxation to numerous substances, including bradykinin, is impaired in arteries isolated from women with pre-eclampsia. The combination of these two findings may contribute to the relative vasoconstriction associated with pre-eclampsia.

Question 8: Liver function: a) Normal pregnancy is associated with a decrease in maternal alkaline phosphatase. b) During normal pregnancy maternal levels of alanine transaminase (ALT) and aspartate transaminase (AST) fall. c) The principal pathological finding in the liver in pre-eclampsia comprises of periportal fibrin deposition, haemorrhage and hepatocellular necrosis. d) An increase in maternal levels of transaminases in pre-eclampsia reflects hepatocellular damage. e) Lactate dehydrogenase levels are a reliable marker of haemolysis. Answer: F, T, T, T, T The relative haemodilution of normal pregnancy leads to an approximate 20% reduction in the level of circulating liver enzymes in pregnancy, with the exception of alkaline phosphatase, which normally increases with increasing gestation. Abnormal liver function in pre-eclampsia reflects liver dysfunction occurring as a result of vasoconstriction of the hepatic bed. Elevated levels of transaminases reflect heptocellular necrosis. Haemolysis may increase asparate transaminase, but will disproportionately increase lactate dehydrogenase levels with serial measurements providing a reliable indicator of the degree of haemolysis present. The histopathology of the liver in pre-eclampsia comprises peri-portal fibrin deposition, haemorrhage and heptocellular necrosis. It is thought that segmental hepatic vasospasm leads to localized coagulopathy allowing fibrin deposition while endothelial and liver-cell necrosis produce haemorrhage. Question, answer and discussion are taken from Self-assessment questions: The pathogenesis of pre-eclampsia, by L.C. Kenny, Current Obstetrics & Gynaecology (1999) volume 9 issue 4

Page 87: Long case examination for phase iii medical students usmkk

In Pursuit to Excel MCQ Exam for Professional III Examination (MCQ)

30

Question 7: Changes in the coagulation system in pre-eclampsia include: a) An increase in maternal levels of antithrombin III. b) An increase in maternal levels of factor VIII related antigen. c) A decrease in maternal levels of fibrinopeptide A. d) A decrease in maternal levels of protein C. e) An increase in maternal levels of protein S. Answer: F, T, T, T, F In normal pregnancy, the overall state of the coagulation system is one of activation. Evidence for this comes from studies which report increased concentrations of clotting factors and raised levels of highmolecular- weight fibrinogen complexes. Sensitive studies of coagulation factors provide evidence that pre-eclampsia accentuates a state of hypercoagulability already created in normal pregnancy. During normal pregnancy, the levels of factor VIII coagulation activity and factor-VIII-related antigen show a proportional rise and, thus, their ratio remains constant. In pre-eclampsia, there is an early rise in the factor- VIII-related antigen:coagulation activity ratio which correlates with the severity of the disease. This increased ratio is a result of increased levels of factor- VIII-related antigen. This substance is synthesized by endothelial cells and megakaryocytes and is released by aggregating platelets. It, therefore, seems probable that the increase in circulating levels is a result of endothelial damage and platelet aggregation. The action of thrombin on fibrinogen is a crucial step in the coagulation cascade. Thrombin cleaves two pairs of peptides, fibrinopeptides A and B from fibrinogen to produce fibrin. Fibrinopeptide concentrations are considered to be the best markers of accelerated thrombosis. The majority of studies have reported increased

levels of fibrinopeptides in women with pre-eclampsia as compared with normal pregnant women. Anti-thrombin III is produced by the liver and is an important inhibitor of coagulation. Levels are unchanged in normal pregnancy but are decreased in the majority of women with pre-eclampsia. The decline in anti-thrombin III activity is thought to result from increased consumption and has been reported to precede the development of clinical signs by as much as 13 weeks. Protein C is a potent inhibitor of activated factor V and VIII, and is an activator of fibrinolysis. Levels are substantially reduced in pre-eclampsia. Activated protein C resistance resulting from a mutation in coagulation factor V has now emerged as a leading cause of thrombosis in pregnancy and a recent report links the HELLP syndrome with factor V mutation. Protein S serves as a cofactor for activated protein C. Levels of protein S in pregnancy may decrease to levels similar to those with congenital protein S deficiency. Furthermore, levels of protein S decrease further in women with pre-eclampsia, as compared with normal pregnant controls. Question, answer and discussion are taken from Self-assessment questions: The pathogenesis of pre-eclampsia, by L.C. Kenny, Current Obstetrics & Gynaecology (1999) volume 9 issue 4

Page 88: Long case examination for phase iii medical students usmkk

In Pursuit to Excel MCQ Exam for Professional III Examination (MCQ)

31

Question 6: Concerning renal function in pre-eclampsia: a) The proteinuria of pre-eclampsia results mainly from a loss of intermediate weight proteins. b) The rate of increase in the amount of proteinuria is both a sensitive and specific predictor of maternal outcome. c) Glomerular filtration deteriorates first and is followed by a deterioration in tubular function. d) Hyperuricaemia generally develops before proteinuria. e) Glomerular endotheliosis is present in all cases of established pre-eclampsia. Answer: T, F, F, T, F Renal function deteriorates in pre-eclampsia in two stages. The first stage involves impairment of tubular function and this is reflected by a reduction in uricacid clearance and the development of hyperuricaemia. Later, glomerular filtration becomes impaired and proteinuria of intermediate selectivity develops. An increasing plasma urate is thus an early sign in the evolution of pre-eclampsia and proteinuria is conventionally recognized as a late sign. The presence of proteinuria heralds a poorer prognosis for the fetus, but the actual amount of proteinuria and the rate of increase have been found to be poor predictors of maternal or perinatal outcome. Renal biopsy in established cases of pre-eclampsia demonstrates a characteristic non-inflammatory lesion commonly referred to as glomerular endotheliosis. This primarily involves a combination of swelling of the glomerular endothelial cells and sub-endothelial ‘fibrinoid’ deposits, which encroach on and occlude the capillary lumen. This lesion is characteristic, but not pathognomonic of pre-eclampsia, as normal renal pathology has been described in some cases.

Question 10: Eclampsia: a) Is an absolute indication for delivery. b) Complicates 15 in 10 000 maternities in the UK. c) Can occur in the absence of significant hypertension. d) Leads to death most commonly by cerebral haemorrhage. e) Can result in cortical blindness. Answer: T, F, T, T, T The principal and clinically most disturbing cerebral sequelae of pre-eclampsia are eclamptic convulsions. Eclampsia is defined as the occurrence of one or more convulsions, not attributable to other cerebral conditions in a patient with pre-eclampsia. It should be noted, however, that eclampsia can rarely present in the absence of significant hypertension. The reported incidence varies geographically; figures from the UK suggest the incidence to be 4.9/10 000 maternities. Eclampsia is thought to result from focal cerebral vasospasm and hypoperfusion leading to abnormal electrical activity and seizures. The lack of any neurological deficit following uncomplicated eclampsia supports the concept that vasospasm precipitates the convulsions. This is in contrast to convulsions caused by thrombosis and haemorrhage, as is often the case in postpartum eclampsia, when significant cortical damage may occur, including the development of cortical blindness. The commonest cause of death in women dying with eclampsia is cerebral haemorrhage. Question, answer and discussion are taken from Self-assessment questions: The pathogenesis of pre-eclampsia, by L.C. Kenny, Current Obstetrics & Gynaecology (1999) volume 9 issue 4

Page 89: Long case examination for phase iii medical students usmkk

In Pursuit to Excel MCQ Exam for Professional III Examination (MCQ)

32

Question 9: Regarding drug treatment in pre-eclampsia: a) Angiotensin-converting enzyme inhibitors are not associated with serious side effects in the antenatal period. b) Peak plasma levels of methyldopa are reached 24 h after an oral dose. c) Labetolol is a combined a- and b-adrenoceptor blocker. d) Nifedipine can be safely used concomitantly with magnesium sulphate. e) Hydrallazine has no effect on uteroplacental blood flow. Answer: F, F, T, F, F The hypertension of pre-eclampsia is caused by increased peripheral resistance and drug treatment is directed towards relieving this without compromising cardiac output. Methyldopa has been extensively studied in pregnancy and is the agent of choice for chronic blood-pressure control. It reduces systemic vascular resistance without significantly altering heart rate or cardiac output. Peak plasma concentrations are reached after 2 h, although the maximum fall in arterial pressure occurs 4–8 h after an oral dose. Labetolol is a combined a- and b-adrenoreceptor blocker. The a1 blockade induces vasodilatation with little change in the cardiac output. Nifedipine can be used for acute or chronic treatment and appears to be safe in pregnancy. It lowers systemic vascular resistance and improves cardiac output without adversely comprising uteroplacental blood flow. It is, however, best avoided in combination with magnesium sulphate as profound hypotension may result. Hydrallazine is used for the acute management of hypertensive crises. It too lowers systemic vascular resistance, but it has a variable effect on uteroplacental blood flow and can occasionally lead to fetal distress. Angiotensin-converting enzyme inhibitors are associated with

intrauterine growth retardation, oliguria, renal failure and neonatal anuria in late pregnancy, and are thus contraindicated in pregnancy. Question, answer and discussion are taken from Self-assessment questions: The pathogenesis of pre-eclampsia, by L.C. Kenny, Current Obstetrics & Gynaecology (1999) volume 9 issue 4